torts final

¡Supera tus tareas y exámenes ahora con Quizwiz!

Effect of the doctrine

(1) Proof of an unexcused statutory violation can be conclusive proof of the violator's breach of duty (a) Jury must determine if violation occurred (2) Shows that legal system has inherent respect for the legislative process

A burglar broke into his neighbor's garage, opened the hood of the neighbor's car, and tried to start the engine. He managed to start the engine, but also caused the car to lunge forward. The burglar was pinned between the car and the wall. The gas tank ruptured and gas spilled all over the garage floor. A passerby heard a burglar screaming for help and ran to assist him. A passerby grabbed a crowbar and managed to free the burglar. The crowbar caused severe cuts in the passerby's hand. "Thanks, you saved my life," said the burglar. "You're welcome," the passerby replied, as he lit a cigarette and threw the match on the floor. The match ignited the spilled gas. Both the burglar and the passerby suffered severe burns. If the burglar asserts a claim against the passerby for injuries sustained in the fire, the burglar may:

(A) Prevail, because the passerby was negligent. (A) One has no legal duty to rescue another, but if he does decide to volunteer in a rescue attempt, he must act in the manner of an ordinary, prudent person. In this question, the passerby did not act reasonably when he lit a cigarette and through the match down, and is liable for the damage caused by his negligent behavior.

A tanker truck driver delivering gasoline to filling stations parked on the street in front of a building to get a cup of coffee after his deliveries. Contrary to company regulations, he parked in an area clearly marked as a no parking zone. Just as the driver went across the street, a minor quake jolted the area and caused a collapse of the walls of the building, which was extremely run-down and unstable. The front wall fell on top of the truck, causing gasoline to leak from the truck and stream down the street. A pedestrian walking two blocks away lit a cigarette and tossed a match in the street, causing the gasoline to ignite. The flames spread to a nearby commercial building and an explosion occurred, causing many windows in a neighboring apartment building to be blown inward. Flying glass was propelled into a tenant's apartment. The tenant suffered multiple cuts and a serious eye injury from the flying glass. The tenant brought a negligence action against the tanker company that employed the driver. If the court finds in favor of the tanker company it will be because:

(A) The court follows the majority view regarding unforeseeable plaintiffs. (A) If the tanker company prevails, it will be because the court follows the majority approach to duty questions. Under the majority approach articulated by Justice Cardozo in Palsgraf v. Long Island Railroad, a plaintiff can recover for the defendant's breach of duty only if she can establish that a reasonable person would have foreseen a risk of injury to her in the circumstances; i.e., that she was located in a foreseeable zone of danger. Here, even assuming that the tanker driver breached a duty by parking his truck on the street, the tenant was not within a foreseeable zone of danger and would not be able to recover damages under this view. (B) is incorrect because the tanker driver's stopping for a cup of coffee after making deliveries does not take him outside the scope of the employment relationship. Hence, if the tanker driver breached a duty to the tenant the tanker company would be vicariously liable for her injuries. (C) is incorrect because even if the owner of the unstable building were a legal (proximate) cause of the tenant's injuries, that would not preclude the tanker driver and the tanker company from being a legal cause as well. (D) is wrong because the fact that the tanker driver violated company rules by parking on the street would not prevent the tanker company from being liable for the tanker driver's actions, because he was still acting within the scope of his employment relationship

Alexander suffers a single indivisible injury as a result of the negligence of Byron and Charles in a state that has not modified the rules of joint and several liability. Alexander's loss is $100,000 . He settles with Byron for $30,000 . Alexander then sues Charles. How much will Charles have to pay? (A) $100,000, which is the amount of the judgment that will be awarded against Charles (B) $70,000, the amount of the $100,000 loss that has not already been satisfied by Byron (C) $30,000 because under a system of joint and several liability, Charles's liability cannot exceed Byron's (D) Nothing because the settlement with Byron discharges Charles from liability

(B) $70,000, the amount of the $100,000 loss that has not already been satisfied by Byron

Which of the following is correct regarding self-defense?A Retaliation may be permissible.B Deadly force may be permissible.C Retreat is required.D Actual necessity is required

(B) Deadly force may be permissible for self-defense. A person may use deadly force to prevent death or serious bodily injury to herself. Self-defense requires use of force that reasonably appears necessary to prevent harm. A person may use deadly force if she reasonably believes that she is in danger of serious bodily injury. Retaliation is not permissible for self-defense. Self-defense is only permitted to prevent the commission of a tort. A person cannot retaliate by using force when there is no longer a threat of injury. Retreat is not required for self-defense. A majority of courts hold that a person may stand her ground and need not attempt an escape. Actual necessity is not required for self-defense. A person need only have an apparent necessity to defend oneself, i.e., a reasonable belief that she is being, or is about to be, attacked.Thus, a reasonable mistake as to the need for self-defense does not eliminate this defense

A private company owns and operates a subway system. The subway has a staff of safety engineers who examine the system to ensure the passengers' safety. For the past seven years, the system has been using the X-1, an air-conditioned subway car complete with the latest technological advances. A forty-year-old blind woman has been riding the subway to work for almost twenty years. When the train pulls into her station she waits for the doors to open and then pokes for the opening with her walking cane. Upon feeling an opening, she steps onto the train. One morning, a train made of X-1 cars pulled into the blind woman's station. As always, the blind woman felt with her cane. The design of the X-1 left a space as large as an open door between the cars. The blind woman mistook the space for an open door and stepped forward. She fell to the tracks and received severe electrical burns from the third rail. A subsequent investigation revealed that twelve blind people had made the same mistake and fallen to the tracks. If the blind woman asserts a claim against the private company, she will most likely:

(B) Recover, if the private company could have taken reasonable steps to prevent blind persons from walking onto the tracks. (B) If the private company had notice of the danger (by virtue of the past twelve occurrences) and could have taken reasonable steps to prevent the injury, the blind woman will most likely prevail in an action for negligence. (A) is incorrect because although common carriers are generally held to a high standard of care, but they are not strictly liable and they are not insurers for all individuals on their premises. Posting barriers between cars and/or warning blind commuters are examples of such measures. (C) is incorrect because "assuming the risk" is rarely the correct answer. Comparative negligence statutes have made assumption of the risk considerations largely unnecessary. (D) is incorrect for the same reasons that (B) is correct.

Which of the following could not constitute battery if done with the requisite intent? (A) Startling the plaintiff so that she falls down stairs (B) Shouting in the plaintiff's ear (C) Deliberately blowing smoke in the plaintiff's face (D) Grabbing the plaintiff's camera, which is hanging around her neck

(B) Shouting in the plaintiff's ear b). All of the others involve unwanted physical contact of some kind, which is an essential element in battery.

A famous comedian was asked by the host of a popular late night television talk show what brand of cigars he smoked. He responded, "I smoke only [the manufacturer's] cigars, because they're the best."Two weeks later the manufacturer of those cigars began a national advertising campaign featuring billboards, posters for use in retail stores, and full-page ads in high circulation magazines. The advertising featured a picture of the comedian with the manufacturer's cigar in his hand, and the copy quoted his statement from the show. The manufacturer had not received the comedian's permission to use either his picture or the statement that had been made during the interview.Will the comedian prevail in an action against the manufacturer for using his picture and state- ment?(A) Yes, because the comedian has been de- famed.(B) Yes, because the comedian's likeness was appropriated for a commercial purpose without his consent.(C) No, because the advertising accurately reflects what the comedian said publicly before millions of television viewers.(D) No, because the comedian's appearance on television created an implied consent to reasonable use of anything he might say

(B) The comedian will prevail on invasion of privacy grounds. (B) is a correct statement of the law defining the invasion of the comedian's privacy by appropriating his likeness for a commercial purpose (i.e., using the comedian's picture to promote the sale of a commercial product without the comedian's consent). (A) is incorrect because the comedian lacks a prima facie case for defamation (there was no defamatory statement causing damage to the comedian's reputation) and, in any case, what the manufacturer printed was true. (C) is relevant to defamation, but is incorrect here, because accuracy is not a defense to privacy torts. (D) is incorrect because consent to commercial appropriation must be specific; it may not be implied from the comedian's state- ments on the television program

A tenant moving into a new apartmentbought a spool of "10-pound test" fishing line, manufactured by a fishing tackle and accessories company, for the purpose of hanging pictures, all of which she knew weighed less than 10 pounds. The spool came with no guidelines or warnings about using it for hanging objects. She attached the fishing line to either end of the pictures and hung them on hooks on the wall. The next week, a friend visiting the tenant was hit and injured while sitting on the couch by a picture thatfell when the fishing line broke. It is common knowledge in the sporting goods industry that "10-pound test" indicates that the line willstand a pull of 10 pounds, but is not intended to support a 10-pound weight over a period of time. However, it is also common knowledge in the industry that the public in general uses fishing line to support heavy hanging objects over a period of time.The friend brought a products liability action against the tackle company for damages caused by his injury.What is the most likely result?(A) The friend will win, because the line failed to support a weight of less than 10 pounds.(B) The friend will win, because the label did not warn the consumer against relying on the term "10-pound test" for purposes other than fishing.(C) The friend will lose, because the line was not being used for its intended purpose, fishing.(D) The friend will lose, because the line conformed to the accepted standard for "10-pound test line."

(B) The most likely result is that the friend will prevail on grounds of inadequate warnings. Where a product presents an unreasonable risk of injury to users and bystanders, the fact that there is no sufficient warning of the danger may be a dangerous defect that will invoke strict products liability. The facts establish that the fishing tackle company knew that purchasers of its fishing line commonly used that line to support heavy hanging objects, and that the line would not support the weight rating in that type of use. Thus, the fishing tackle company's failure to provide a warning may be a defect that will permit the friend to recover in strict liability. Given that the other choices are clearly incorrect, (B) is the best option. (A) is incorrect because the fact that the line would not support a hanging weight of less than 10 pounds is not alone sufficient to constitute a dangerous defect; it is the fact that the fishing tackle company knew of the danger represented and failed to provide a warning to such users that may make the injury to the friend actionable. (C) is incorrect because the foreseeable misuse of a dangerously defective product by a purchaser or user does not relieve the manufacturer of the product of liability for injuries that are caused by the defect. Here, the known use of fishing line to support heavy hanging weights is what prompts the need for a warning, the absence of which may make the product dangerously defective. (D)is incorrect because, as indicated above, the presence or absence of a dangerous defect, and the responsibility for injuries caused by the defect, is not controlled by whether a product meets standards of performance measured solely by the use intended by the manufacturer. The fact that the fishing line performed adequately under industry standards when used for fishing does not mean that it cannot be dangerously defective when used for another purpose, when such use is known or reasonably should have been known by the manufacturer.

A teenager who was totally blind in one eye and had only 10% vision in the other couldnot obtain a driver's license. Nevertheless, onhis 18th birthday, he borrowed his father's car and took his girlfriend for a ride. With his 10% vision in one eye, he was able to stay in the correct lane and avoid oncoming traffic, but he failed to see a jogger on the edge of the highway. The teenager's car hit the jogger, causing serious bodily injury.If the jogger brings a negligence suit against the teenager and the jury finds in the jogger's favor, what is the most likely reason?(A) The teenager failed to exercise ordinary and reasonable care under the circumstanc- es.(B) The teenager failed to exercise the amount of care that an 18-year-old of like educa- tion, intelligence, and experience would have exercised.(C) The teenager failed to exercise the ordinary and reasonable care that a person with the teenager's disability would have exercised.(D) Theteenagerviolatedthelawwhenhe drove without a license

(C) If the jogger prevails, it will be based on failure to exercise ordinary care of someone with the teenager's vision problem. In a lawsuit based on negligence, the usual standard of care is ordinary and reasonable care under the circumstances. The standard changes when the defendant has a major physical disability such as blindness. In that situation, the standard becomes ordinary and reasonable care for a person with that disability. Therefore, (C) is a more accurate answer than (A). (B) is wrong because it states the standard to be applied to children, which would not be applied to an 18-year-old car driver. (D) is a true statement. However, the statutory violation of failure to obtain a license was not the cause of the accident

Which of the following must the plaintiff prove as an element of the tort of battery?

(C) That the defendant intended to cause harmful or offensive contact with the plaintiff or realized that it was likely to occur (c). It is not necessary for the defendant to have intended to harm, injure or offend the plaintiff. It is not enough merely to intend the action that caused the contact. The defendant must have intended the action and the consequence that it would make harmful or offensive contact. It is enough if the defendant intended the action and realized that harmful or offensive contact was likely to occur; that is why answer (b) is not correct, as that intention is not essential in all cases.

A pedestrian crossed the street at a crosswalk without looking for oncoming traffic. He was struck first by a car and then by a truck. The pedestrian sued both the driver of the car and the driver of the truck for negligence. The jury determined that the pedestrian was 60% at fault, the driver of the car 30%, and the truck driver 10%. The jury also determined that the pedes- trian suffered damages of $100,000. The driver of the car is insolvent.In a pure comparative negligence jurisdiction retaining traditional joint liability rules, how much can the pedestrian collect from the driver of the truck?

(C) The pedestrian can collect $40,000 from the driver. In a pure comparative negligence jurisdic- tion, the plaintiff can recover even if he was over 50% at fault. Thus, (A) is wrong. The recovery will be limited to the percentage of damage attributed to the defendant(s), in this case 40%. The jurisdiction retains the rule on joint and several liability. Therefore, each defendant is responsible for the combined liability of all defendants. (C) is therefore correct and (B) is wrong. (D) is wrong because, in a comparative negligence jurisdiction, the plaintiff's recovery is reduced due to his negligence

While returning from transporting a groupof children to summer camp, a bus driver and his assistant were caught in the leading edge of a forest fire raging down the high mountains. Hurrying ahead of the flames and smoke, the driver reached the last half-mile of a dirt road that ran to the main highway and safety, buthe discovered that the road ahead was already blocked by fallen, burning foliage. Separating the driver's bus from the main highway, which angled off to the right, was the fenced property of a rancher. The bus driver drove across the property to reach the main highway, damaging some turf and a fence, and proceeded to the city.If the rancher asserts a claim against the bus driver to recover for the damage to his property, is the rancher likely to win?(A) No, because the bus driver was acting to protect the lives of himself and his assis- tant.(B) No, because the bus driver acted as would any reasonably prudent person under the circumstances.(C) Yes, because the bus driver damaged the rancher's property when he drove through the fence to get to the main highway.(D) Yes, because the bus driver intentionally drove across the property, knowing it would cause damage

(C) The rancher will probably win. A person who intentionally intrudes upon land in the possession of another is guilty of the tort of trespass to land. The bus driver has clearly met all the elements of this tort. However, the bus driver has the defense of private necessity, because it was necessary to drive onto the rancher's land to avoid the forest fire. Although private necessity is a defense to trespass to lands, it does not relieve the bus driver of liability for damage done to the property. Hence, (A) and (B) are incorrect. Note that the call of the question merely asks what the outcome of the rancher's claim against the driver would be, not whether the driver has committed a trespass. (D) is not as good a choice as (C) because it focuses on the driver's intent in committing a trespass rather than on whether the land was damaged

On a snowy night, a driver was driving his pickup truck down a state road. He happened to notice a car parked strangely on the side of the road and decided to investigate. Five severely inebriated people were sleeping in the car. The driver opened an unlocked door and tried unsuccessfully to wake the car's occupants. Fearing they would die of exposure, the driver locked his own truck, which was too small to transport the drunks, and started the parked car. He shifted the car into reverse. The driver was unaware that the car's transmission was not working and the car lurched forward instead of going backward, causing it to slide down a ravine. All the occupants suffered serious injuries. If the passengers assert a claim against the driver, they will most likely:

(D) Not recover, if the driver acted reasonably. Correct D) The driver was most likely correct in determining that the passengers were in danger. In general, one has no duty to take affirmative action to benefit others. However, one who does act is under the duty to act in the manner of a reasonably prudent person. The driver acted properly if he reasonably assumed he could drive the passengers to safety. (A) is incorrect because under ordinary circumstances the driver's actions would not have placed the passengers in any greater danger.(B) and (C) are incorrect because they are just not relevant to the question. Look for answers that do not deal with the elements of the tort being asserted, as they are sometimes easy to eliminate.

A business owner is in the business of installing and reconditioning elevators. A statute, enacted for safety reasons, provides that elevators must have a safety device to absorb the impact caused when an elevator travels too fast toward the bottom of a shaft. The business owner regularly purchases this safety device from a retailer. The business owner conducts its own tests on the safety devices. A company contracted with the business owner to install six new elevators in a tower that the company recently purchased and refurbished.An employee of the company operated the tower's freight elevator, which was also installed by the business owner and is identical to the other elevators in the building, except that it must be manually operated. On one occasion, the employee negligently let the elevator drop to the basement at high speed. The safety device, which should have prevented the passengers from being injured, failed. Two passengers suffered serious injuries, as did the employee. The employee would have suffered the same injuries even if the safety device had worked properly, because he had been leaning on the drive shaft. Subsequent tests showed that the metal in the safety device was defective. The business owner had performed these same tests before installing the elevator. If the passengers bring a negligence action against the business owner, to establish a prima facie case it is necessary that they prove that:

(D) The safety device was defective and would have been discovered had the business owner exercised reasonable care. (D) The elements of a prima facie case of negligence are (1) a duty, (2) a breach of the duty, (3) causation, and (4) damages. It should be clear that the installer of a safety device has a duty to diligently inspect it. Breach of this duty (D) remains to be proved. The proximate cause of injury, for the purposes of this action, is the failure of the safety device (A) which is not in dispute. (B) is incorrect because mere failure of the business owner to inspect will not be enough to prove that he breached his duty. (D) is enough to prove he breached his duty, since the answer goes further and speaks of reasonable care. (C) is incorrect because had he taken reasonable measures to inspect, and still not discovered it (but the inspection was what a reasonable person would do), that would still not establish a breach of the duty.

Implied assumption of risk is usually divided into two sub-categories: primary and secondary implied assumption of risk. Which of the following most accurately describes what "primary implied assumption of risk" means? (A) The plaintiff voluntarily accepted the risks inherent in the activity in question and so the defendant did not breach any duty of care owed to the plaintiff in respect of that activity (B) The plaintiff voluntarily accepted the characteristic risks of the activity in question but not any secondary or incidental ones (C) The plaintiff's assumption of risk defense is of primary importance, so the defense prevails over apportionment of liability because of comparative fault (D) The plaintiff was aware of and voluntarily accepted the risk of negligent conduct on the part of the defendant

(a). "Primary implied assumption of risk" refers to assumption of the risks inherent in the activity provided by the defendant. Because those risks are inherent, the defendant owes no duty in relation to them. For that reason, some writers say that the language of assumption of risk should be abandoned in cases such as these.

Question 1 Alexandra negligently injures Bella. Bella becomes extremely depressed as a result of the injuries she suffers and she eventually commits suicide. Which of the following would not be a possible outcome if Bella's estate were to sue Alexandra? (A) Bella's estate must lose because Alexandra did not kill Bella, Bella did (B) Bella's estate will probably lose because suicide is generally an unforeseeable intervening cause (C) Bella's estate might win if Alexandra's negligence caused Bella to suffer mental disturbance destroying the will to survive (D) Bella's estate would win if there were something about the circumstances to indicate that suicide was a normal response to the situation created by Alexandra's negligence

(a). Although suicide claims usually fail for the reasons indicated in answer (b), they can succeed in some circumstances, as answers (c) and (d) correctly indicate. See, e.g., Fuller v. Preis, 35 N.Y. 2d 425, 322 N.E.2d 263 (1974). Because answer (a) states that the claim by Bella's estate could never succeed, it is wrong, and so is the "correct" answer.

Adam bought a staple gun manufactured by BaBoom, Inc. He tested it by putting it up against his arm and firing a staple. The staple punctured an artery and Adam suffered severe blood loss. He sued BaBoom, Inc. in a state where the products liability law is modeled on Section: 402A of the Restatement (Second) on Torts, arguing that the staple gun should have carried a warning against using it on skin or flesh. Which of the following most accurately states the position in relation to BaBoom's liability for failing to warn of the danger posed by the staple gun? (a) BaBoom, Inc. need not put a warning on the staple gun because a product with open and obvious dangers effectively carries its own warning (b) BaBoom, Inc. need not put a warning on the staple gun because it would not be reasonable to expect a manufacturer to warn against an open and obvious danger (c) BaBoom, Inc. should have put a warning on the staple gun because it was foreseeable that someone might fail to recognize an open and obvious danger (d) BaBoom, Inc. should have put a warning on the staple gun because the danger it posed was not open and obvious

(a). Answer (b) uses language of reasonableness but § 402A liability is strict. Answer (c) is not correct; a manufacturer does not have to warn those who cannot see the obvious. Answer (d) seems unlikely given the function of a staple gun, of which Adam was obviously aware.

Anita is injured in an accident caused by the negligence of Barbara and Christine. Anita suffers $100,000 worth of harm. She sues Barbara but not Christine in a jurisdiction that has not amended the rules governing joint and several liability and which has legislation modeled on the Uniform Contribution Among Tortfeasors Act. Barbara wants to seek a contribution from Christine. What must Barbara prove? (A) That Christine would have been liable to Anita if Anita had sued Christine (B) That Christine breached a duty of care to Barbara (C) That Barbara was not wholly responsible for Anita's injuries (D) That Barbara promised Christine (expressly or impliedly) that she would contribute to Barbara's liability to Christine

(a). Answers (b) and (d) are concerned with liability of Christine to Barbara, which has nothing to do with Christine's obligation to contribute to Barbara's liability to Anita. Answer (c) is closer to being right, because Barbara will have established that she was not wholly responsible if she proves that Christine would have been liable to Anita, but it is not correct because it would not be sufficient to establish a right of contribution.

In order to establish the tort of assault, what type of apprehension must the plaintiff prove that he or she had as a result of the defendant's conduct? (A) Well-founded apprehension of imminent contact by the defendant that would amount to battery (B) Well-founded apprehension of future contact by the defendant that would amount to battery (C) Acute personal apprehension of imminent contact by the defendant that would amount to battery (D) Acute personal apprehension of future contact by the defendant that would amount to battery

(a). Answers (c) and (d) depend upon the plaintiff's subjective apprehension, which is not material because an objective test is used. Answer (b) is incorrect because the apprehension must be of some contact in the immediate future, not some unspecified time in the future.

Aaron is severely injured by Brandon's negligence. Aaron has been in a coma since the accident and remains so at the trial of his action against Brandon. Which of the following statements most accurately states what will happen at trial in relation to the award of damages for pain and suffering? (a) Aaron will recover no damages at all for pain and suffering because he has no subjective experience of pain or suffering (b) Aaron will recover substantial damages for pain and suffering to reflect the fact that he has been deprived of consciousness altogether (c) Aaron will be awarded pain and suffering damages that will be put into a trust fund and returned to Brandon if Aaron does not recover consciousness (d) The court will adjourn consideration of pain and suffering damages until it is clear whether Aaron will ever recover consciousness

(a). Damages for pain and suffering are awarded on an entirely subjective basis, so (a) is the correct answer and (b), which depends on an objective assessment, is not. Answers (c) and (d) sound ingenious but they will not happen. The court makes an assessment of plaintiff's damages once and forever at the time of the trial.

Anna is injured in an accident caused by the negligence of Belle and Carol. The jury finds that Belle is one-third responsible for Anna's injuries, Carol is one-third responsible and Anna is one-third responsible for her own injuries. What proportion of her loss will Anna recover in a state that has a modified ("not as great as") system of comparative fault and which does not aggregate multiple defendants' fault for the purposes of comparison? (A) 0% (B) One-third (C) Two-thirds (D) 100%

(a). In a state that does not aggregate, Anna's share of responsibility would be compared with Belle's and Carol's separately and independently. Because her responsibility is as great as (in fact, exactly as great as) that of each defendant, she would recover nothing from either. minority rule

Alice is injured by Brent's negligence. Her employer continues to pay her in full, even when she is off work, because it has a generous sick leave policy. Which of the following most accurately states what will happen if Alice sues Brent? (a) The amount of damages Alice recovers will not be reduced to take into account the fact that she is still being paid by her employer (b) Alice will recover no damages for loss of earnings during the period when she was paid in full (c) Alice will only be able to recover loss of earnings damages from Brent for the period after her sick pay runs out

(a). The collateral benefit or collateral source rule applies to sick pay as well as to insurance benefits.

Brian is knocked from his motorcycle by the negligent driving of Charles, who crashes his car into Brian's motorcycle. Danielle, a pedestrian passing by just feet away from the impact, witnesses the accident. Brian suffers horrific injuries. About ten minutes later, Ann, Brian's wife, returns to her home from a short visit to the local store. She sees Brian lying bleeding in the street. Both Ann and Danielle suffer acute distress and recurring psychological problems. What would the likely outcome be if Ann and Danielle sued Charles in a state that has abandoned the traditional restrictions on recovery and uses ordinary principles of foreseeability to determine liability in cases of negligent infliction of emotional harm? (a) Ann and Danielle would both win (b) Ann would win but Danielle would lose (c) Ann would lose but Danielle would win (d) Ann and Danielle would both lose

(a). This answer may be obvious but it is included to contrast the outcome with the variants in Questions [73] to [77].

Alberto likes mushrooms. He buys the book The Encyclopaedia of Mushrooms and goes mushroom hunting, relying on the descriptions in the book. He picks, cooks and eats some wild mushrooms, which turn out to be poisonous. Alberto gets acute food poisoning. He sues the publisher of the encyclopaedia in a state where the products liability law is modeled on Section: 402A of the Restatement (Second) on Torts. Which of the following most accurately states the likely outcome? (a) The publisher is not liable because the information in the book is not a "product" for the purposes of strict liability under Section: 402A (b) The publisher is not liable under Section: 402A because it could not reasonably have expected that readers would fall ill as a result of relying on the information in the book (c) The publisher is strictly liable under Section: 402A because the information in the book was a defective product for the purposes of Section: 402A (d) The publisher is strictly liable under Section: 402A because the book is a product and the book caused Alberto harm

(a). This question is based on Winter v. G.P. Putnam's Sons, 938 F.2d 1033 (9th Cir. 1991)), where the court held that strict products liability does not extend beyond tangible items. Alberto might be able to succeed if he gashed his fingers on the staples holding the book together, but the information in the book is not a product.

Alexander suffers a single injury as a result of separate acts of negligence by Byron and Charles in a state that has not modified the rules of joint and several liability. Which of the following statements is correct? (A) Alexander may sue either Byron or Charles and can recover damages in full from either (B) Alexander must sue both Byron and Charles in order to recover damages in full (C) If Alexander sues only Byron, Byron has a partial defense that he was not solely to blame for Alexander's injuries (D) If Alexander sues only Charles, Charles cannot be held liable for a greater percentage of Alexander's loss than Charles's proportion of responsibility

(a). Under a system of unmodified joint and several liability, each person causing harm is liable in full for the harm caused by all. Under such a system, it is no defense to say that others also caused the plaintiff's harm.

Anna is injured in an accident caused by the negligence of Belle, Carol and Diana. The jury finds that Belle is 30% responsible for Anna's injuries, Carol is 20% responsible, Diana is 25% responsible and Anna is 25% responsible for her own injuries. What percentage of her loss will Anna recover in a state that has a modified ("not as great as") system of comparative fault that does not aggregate fault on the defendants' part, but which has also adopted a system of several (not joint and several) liability? (A) 0% (B) 30% (C) 75% (D) 100%

(b). Anna would only be able to recover from Belle, because only Belle's responsibility is greater than her own. The basic rule of several liability is that no person should bear a greater proportion of the loss than her or his share of responsibility, so Belle's liability cannot exceed 30%. The comparative fault rules thus serve to insulate Carol and Diana from liability without shifting any of the burden to Belle.

Alexander and Alexandra have consensual sex. Alexander knows that he has the sexually transmitted disease genital herpes but he does not tell Alexandra. She later contracts herpes. Which of the following most accurately states the likely outcome if Alexandra sues Alexander in the tort of battery? (A) Alexandra's action will fail because she consented to having sex with Alexander (B) Alexandra's action will succeed because she would not have consented to the contact she had with Alexander if she had known he had herpes (C) Alexandra's action will succeed because Alexander concealed the fact he had herpes (D) Alexandra's action will fail because sexual contact cannot amount to battery

(b). Answer (a) is incorrect because Alexandra's consent arose out of a mistake induced by Alexander's silence. Mistaken consent cannot authorize contact that would otherwise constitute battery. Answer (d) is wildly wrong: sexual contact can most definitely amount to battery if it occurs without consent. Answer (c) is correct but incomplete; it does not explain why Alexander's concealment of his herpes will be relevant. See, e.g., Kathleen K. v. Robert B., 198 Cal. Rptr. 273 (Cal. App. 2 Dist. 1984); Doe v. Johnson, 817 F. Supp. 1382 (W.D. Mich. 1993).

Most states that have abolished the complete defense of contributory negligence in favor of the comparative fault system of apportionment of liability have also abolished the complete defense of secondary implied assumption of risk. Which of the following statements most accurately describes the reason for abolishing the assumption of risk defense in a jurisdiction that has moved to comparative fault? (A) Having two different defenses with different effects (comparative fault a partial defense, assumption of risk a complete defense) is too complicated for a jury to understand (B) The plaintiff's conduct in voluntarily assuming a known risk amounts to a negligent failure to take care for her or his own safety, which should lead to apportionment of liability under a comparative fault system, not the complete failure of the plaintiff's action (C) Because secondary implied assumption of risk also amounted to contributory negligence, abolition of the latter defense necessarily indicated abolition of the former (D) The defense of contributory negligence was abolished in order to reduce the number of cases that would be put to the jury; that goal would be undermined if the complete defense of secondary implied assumption of risk were to be retained

(b). Answer (a) may be true but it is not the reason why the complete defense has been abolished in most comparative fault states. Answers (c) and (d) might sound plausible but neither is true.

Ann kisses her husband Brian goodbye as he prepares to go to work on his motorcycle. Ann steps back and is immediately horrified to see Brian knocked from his motorcycle by the negligent driving of Charles, who crashes his car into Brian's motorcycle. Danielle, a pedestrian passing by on the other side of the street, witnesses the accident, too. Brian suffers horrific injuries. Both Ann and Danielle suffer acute distress and recurring psychological problems as a result of witnessing the accident. What would the likely outcome be if Ann and Danielle sued Charles in a state that uses the principles established in Dillon v. Legg, 441 P.2d 912 (Cal. 1968) to determine liability in cases of negligent infliction of emotional harm? (a) Ann and Danielle would both win (b) Ann would win but Danielle would lose (c) Ann would lose but Danielle would win (d) Ann and Danielle would both lose

(b). Dillon v. Legg allows recovery to a plaintiff who: (1) is near the scene of the accident; (2) suffers shock from a direct emotional impact as a result of contemporaneous sensory observance of the accident, and (3) is closely related to the victim who suffers physical harm. Ann satisfies all three requirements. Danielle satisfies (1) and (2) but not (3).

Alexa waves at Bella in friendly fashion and reaches out to pat her on the shoulder. Bella, who has a pathological fear of catching germs from others, recoils violently from the contact. Which of the following is correct? (A) Alexa has assaulted Bella because Bella reasonably feared contact that she (Bella) would find offensive (B) Alexa has not assaulted Bella because the contact that Alexa threatened would not have been regarded as offensive by a reasonable person (C) Alexa has not assaulted Bella because Bella was abnormally apprehensive that Alexa would make contact with her (D) Alexa has assaulted Bella because Bella actually feared contact that a reasonable person would find offensive

(b). Plaintiff must show contact that would offend a reasonable sense of personal dignity, which makes (a) the wrong answer because it relies on B's subjective notion of offense. It seems clear that (d) is wrong because a reasonable person would not find this kind of contact offensive. Answer (c) sounds like it might be right, but B's abnormal sensitivity lies not in her fear that A will make contact with her.

Anna is injured in an accident caused by the negligence of Belle and Carol. The jury finds that Belle is 30% responsible for Anna's injuries, Carol is 30% responsible and Anna is 40% responsible for her own injuries. What percentage of her loss will Anna recover in a state that has a "pure" system of comparative fault? (A) 40% (B) 60% (C) 0% (D) 100%

(b). The fact that Anna's share of responsibility is greater than either Belle's or Carol's would be irrelevant in such a state. If the rules of joint and several liability still apply, Anna would get judgment for 60% of her loss from both Belle and Carol.

Annabelle keeps her pet panther Boris in a pen in her back yard. She has had Boris since he was a small cub. Boris is very tame. One day, Boris gets out of his pen and crosses the road into Catherine's yard. While there, Boris steps on the remote control for the garage door, which Catherine has left lying in the yard. The garage door closes, damaging Catherine's car, which is parked half-way into the garage. Which of the following answers is most accurate? (a) Because a panther is a wild animal, Annabelle is strictly liable for all harm done by Boris (b) Because stepping on remote controls is not one of the propensities that make panthers dangerous, Annabelle is not strictly liable for the harm done by Boris (c) Because Boris is tame, he would be considered a household pet, so Annabelle's liability would not be strictly liable for harm done by Boris (d) Because a panther is not a farm animal, Annabelle is not strictly liable for any damage he might do if he escapes

(b). There is only strict liability for wild animals if they cause the kind of harm that falls within their dangerous propensities, so answer (a) is incorrect. Answer (c) is incorrect: a panther is a wild animal for the purposes of legal liability, however tame in fact. Answer (d) is incorrect: farm animals are not the only kind of animals for which there is strict liability.

Which of the following most accurately states the "firefighter's rule" in relation to injuries sustained by a professional firefighter when responding to an emergency? (A) The defense of voluntary assumption of risk never applies to professional firefighters, who have no choice but to assume the risk of injury when responding to an emergency (B) A professional firefighter cannot recover damages for harm suffered while responding to an emergency, even when that harm is caused by the defendant's negligence (C) Professional firefighters are entitled to generous awards of damages for harm suffered while responding to an emergency, in order to encourage people to become firefighters (D) A professional firefighter is entitled to recover punitive damages if he or she is injured while fighting a fire negligently caused by the defendant

(b). This answer may seem surprising, but the policy underlying it is that professional firefighters are paid to undertake the risks they face when responding to emergencies, which are often caused by the negligence of others. To allow them to recover damages as well would be to compensate them twice, once in their wages and once by damages. Because this is the rationale of the rule, it does not apply to volunteer firefighters, who are not paid. Roberts v. Vaughn, 459 Mich. 282, 587 N.W.2d 249 (1998).

Antonia wants to take up water-skiing. Before her first lesson, with Bright Ski Classes, Inc., she is asked to sign a contract that includes the following clause: "I recognize that water-skiing is a dangerous sport. By signing this contract I expressly assume all risk of injury occurring while skiing, whether caused by negligence of Bright Ski Classes, Inc., its employees, or other parties." Antonia is injured during her lesson when the driver of the tow-boat tows her too close to some moored yachts and she is unable to avoid crashing into one. Antonia sues Bright Ski Classes, Inc., which raises the defense of express assumption of risk. Which of the following most accurately states what is likely to happen in relation to Bright's assumption of risk defense? (A) Bright's defense will fail because standard form exclusion clauses in printed contracts are never effective, even if signed (B) Bright's defense will fail because the contractual clause did not cover the risk that harmed Antonia (C) Bright's defense will succeed unless Antonia was deceived into signing the contract or the clause was not adequately brought to her attention (D) Bright's defense will succeed because Antonia signed the contract

(c). Although exclusion clauses in contracts of adhesion may be struck down if the consumer has no real choice — such as, for example, when the consumer is required to sign a waiver before emergency medical treatment or when accepting public housing — answer (a) goes too far in saying that printed standard form clauses never work. The clause did cover the risk of injury caused by negligence, so answer (b) is plain wrong. Answer (d) does not account for the requirement that the consent indicated by signature must be freely given in order to be effective.

Anna is injured in an accident caused by the negligence of Belle, Carol and Diana. The jury finds that Belle is 30% responsible for Anna's injuries, Carol is 20% responsible, Diana is 25% responsible and Anna is 25% responsible for her own injuries. What percentage of her loss will Anna recover in a state that has a modified ("not as great as") system of comparative fault that does not aggregate fault on the defendants' part, but which also has not modified the rules of joint and several liability? (A) 0% (B) 30% (C) 75% (D) 100%

(c). Anna would only be able to recover from Belle, because only Belle's responsibility is greater than her own. Because the state in question has not modified the rules of joint and several liability, Anna could recover the whole of her loss from Belle, reduced by the proportion of Anna's own responsibility (25%). That may seem unfair to Belle, but it is the outcome dictated by the statutes in such states.

Alan was addicted to a violent computer game that required the player to shoot as many people as possible in a short space of time. One day, he armed himself with several handguns and killed many children at the local school, before killing himself. The parents of the victims sued the manufacturer of the video game. Which of the following most accurately states the likely outcome? (A) The manufacturer should lose because it was obvious that players of the game might actually harm others (B) The manufacturer should lose because it was likely that players of the game might actually harm others (C) The manufacturer should win because the risk that players of the game might actually harm others was unforeseeably small (D) The manufacturer should win because Alan killed the children, not the manufacturer

(c). Answer (a) goes too far in stating the risk of harm. Answer (d) might be popular but is not correct. The mere fact that harm was directly caused by the wrongful act of another does not necessarily relieve a tortfeasor of liability. Answer (b) comes closest to being correct, but it surely overstates the case by saying that it was likely that players might actually harm others. Of the four options given, answer (c) is most likely. That has been the outcome in several cases about violent video and computer games. See, e.g., Watters v. TSR, Inc., 904 F.2d 378 (6th Cir. 1990), James v. Meow Media, Inc., 300 F.3d 683 (6th Cir. 2002), Wilson v. Midway Games, Inc., 198 F.Supp.2d 167 (D.Conn., 2002).

Annabelle's sheep Boris strays from Annabelle's paddock into Catherine's yard. While walking around in Catherine's yard, Boris steps on the remote control for the garage door, which Catherine has left lying in the yard. The garage door closes, damaging Catherine's car, which is parked half-way into the garage. Which of the following answers is most accurate? (a) Annabelle is not liable for the damage to Catherine's car because it was not proximately caused by her negligence (b) Annabelle is strictly liable for the harm done by Boris to Catherine's car (c) Annabelle is not liable for the harm done to Catherine's car because it is not the kind of harm reasonably to be expected from Boris's intrusion on Catherine's yard (d) Annabelle is liable for the harm done to Catherine's car but Catherine's recovery is reduced on account of her own fault

(c). Answer (a) raises the right kind of issue, but it speaks of Annabelle's negligence, whereas liability for damage cause by farm animals is strict. Answer (b) would be correct but for the unusual nature of the harm. If Annabelle were liable to Catherine, answer (d) might be correct, but Annabelle would not be liable to Catherine.

Anatole is an alcoholic. He and his wife Beatrice go out for the evening. Anatole drinks heavily all evening until he is very drunk. He then says it is time to drive home. Beatrice tells Anatole that he should not drive their car. Anatole insists that he is capable of driving. Beatrice shrugs and gives him the car keys but she refuses to get into the car with Anatole. On the way home, Anatole runs down Clarence, a pedestrian, severely injuring him. Which of the following statements is most accurate? (a) Beatrice owed Clarence no duty to control the actions of her husband Anatole, who is responsible for his own actions (although he acted irresponsibly) (b) Beatrice owed Clarence no duty to control the actions of her husband Anatole, because she was not riding in the car with him when he ran over Clarence (c) Beatrice owed Clarence a duty to control the actions of her husband Anatole because she created the risk to Clarence by giving Anatole the car keys (d) Beatrice owed Clarence a duty to control the actions of her husband Anatole because she knew that he would not be able to drive safely while drunk

(c). Answer (a) was correct when Beatrice did nothing but it is incorrect when she takes active steps to create risk to others. It makes no difference whether Beatrice is in the car, as answer (b) suggests. Her duty is not to control Anatole while driving. Answer (d) makes the duty to act dependent on Beatrice's knowledge of the risk rather than her creation of the risk, which is incorrect.

Aaron is the punter on his high school football team. Biff, one of the players on the opposing team, runs into Aaron as he is punting the ball. Aaron is injured. Biff's team is penalized 15 yards for roughing the kicker. Which of the following most accurately states the likely outcome if Aaron sues Biff in the tort of battery? (A) Aaron's action will fail because he must be taken to have consented to physical contact, even rough physical contact, by agreeing to play football (B) Aaron's action will succeed because Biff's contact with him was forbidden by the rules of the game (C) Although Biff's contact with Aaron was outside the rules of the game, Aaron must be taken to have consented to it because it was within the range of ordinary contact to be expected in a game (D) Aaron's action will succeed because he did not consent to contact that was not permitted by the rules of the game

(c). Answer (a) wrongly suggests that Aaron must be taken to have consented to any kind of contact, regardless of whether it is forbidden by the rules. Answers (b) and (d) state the same wrong view in different ways. Because it is almost impossible to play any game entirely within the rules, all participants must be taken to have consented to some kind of contact that is not permitted by the rules, but which ordinarily occurs.

Which of the following statements most accurately describes the "Thin skull plaintiff" rule? (A) An unusually sensitive person can recover damages in full for injuries that are greater in extent than would be reasonably foreseeable (B) An unusually sensitive person cannot recover damages for any harm suffered beyond what would have been suffered by an ordinary person (C) An unusually sensitive person can recover damages in full for injuries that are greater in extent than would be reasonably foreseeable if it was reasonably foreseeable that an ordinary person would suffer some kind of harm (D) An unusually sensitive person can only recover damages for injuries that are greater in extent than an ordinary person would suffer if it was reasonably foreseeable that a person with that sensitivity might be harmed

(c). Answer (b) is the opposite of the correct position and answer (d) does not state a special "rule" at all: if the harm suffered by the sensitive person was reasonably foreseeable it would be recoverable in any event. Answer (a) is closest to being correct but it does not contain the important qualification that no damages are recoverable if no harm at all was foreseeable. Unless it is reasonably foreseeable that an ordinary person would suffer some kind of harm, there is no basis for holding the defendant responsible for the harm suffered by the plaintiff.

Question 7 Anatole is an alcoholic. He and his wife Beatrice go out for the evening. Anatole drinks heavily all evening until he is very drunk. He then says it is time to drive home. Beatrice tells Anatole that he should not drive their car and she threatens to take the car keys away from him or to drive the car away herself. Anatole insists that he is capable of driving and he gets into the car. Beatrice shrugs and refuses to get into the car with Anatole. On the way home, Anatole runs down Clarence, a pedestrian, severely injuring him. Which of the following statements is most accurate? (a) Beatrice owed Clarence a duty to control the actions of Anatole because of the special relationship (husband and wife) between Beatrice and Anatole (b) Beatrice owed Clarence a duty to take reasonable steps to prevent Anatole from driving because it would have been easy for her to take the car keys away (c) Beatrice owed Clarence no duty to control the actions of her husband Anatole, who is responsible for his own actions (although he acted irresponsibly) (d) Beatrice owed Clarence no duty to control the actions of her husband Anatole unless she had some reason to believe that he would not be able to drive safely while drunk

(c). Answer (b) makes ease of preventive action the basis for imposition of a duty, which would result in a duty being imposed in many cases, far more than is actually the case. Answer (d) makes knowledge of the likelihood of harm the basis for imposition of a duty, which is also not relevant. Answer (a) sounds plausible, but the relationship of husband and wife, although "special" in some senses, is not a "special relationship" for these purposes because it is not one of custody and control, which is what is necessary to give rise to a duty.

Alan was addicted to a violent computer game that required the player to shoot as many people as possible in a short space of time. One day, he armed himself with several handguns and killed many children at the local school, before killing himself. The parents of the victims sued the manufacturer of the video game. Which of the following conclusions would be necessary before the court could come to the conclusion that the manufacturer should be held responsible for the victims' death? (A) The computer game helped Alan to lose his inhibitions about harming others (B) Alan would not have shot the children if he had not played the computer game (C) The manufacturer should reasonably have foreseen that players might be encouraged to shoot people in real life (D) Playing the game made Alan disconnect violence from its natural consequences

(c). Answers (a), (b) and (d) merely show a "cause in fact" connection between the game and the deaths. That would not be enough in itself to establish liability. The harmful consequences must be a foreseeable consequence of the wrongful act.

Anna is injured in an accident caused by the negligence of Belle and Carol. The jury finds that Belle is one-third responsible for Anna's injuries, Carol is one-third responsible and Anna is one-third responsible for her own injuries. What proportion of her loss will Anna recover in a state that has a modified ("not as great as") system of comparative fault and which aggregates multiple defendants' fault for the purposes of comparison? (A) 0% (B) One-third (C) Two-thirds (D) 100%

(c). In a state that aggregates for the purposes of comparison (as most do), Anna's share of responsibility would be compared with Belle's and Carol's taken together. Because her responsibility is not as great as that of Belle and Carol taken together, she would recover her loss reduced by the amount (one-third) of her own responsibility. majority rule

Anna is injured in an accident caused by the negligence of Belle and Carol. The jury finds that Belle is 30% responsible for Anna's injuries, Carol is 20% responsible and Anna is 50% responsible for her own injuries. What percentage of her loss will Anna recover in a state that has a modified ("not greater than") of comparative fault which aggregates fault on the defendants' part? (A) 0% (B) 20% or 30%, depending on whether she collects from Belle or Carol (C) 50% (D) 100%

(c). In a state that aggregates for the purposes of comparison (as most do), Anna's share of responsibility would be compared with Belle's and Carol's taken together. Because her responsibility is not greater than that of Belle and Carol taken together (because it is equal to it), she would recover her loss reduced by the amount (50%) of her own responsibility.

Anna is injured in an accident caused by the negligence of Belle and Carol. The jury finds that Belle is 30% responsible for Anna's injuries, Carol is 30% responsible and Anna is 40% responsible for her own injuries. What percentage of her loss will Anna recover in a state that has a modified ("not greater than") system of comparative fault and which does not aggregate multiple defendants' fault for the purposes of comparison? (A) 40% (B) 60% (C) 0% (D) 100%

(c). In such a state, Anna's share of responsibility would be compared with Belle's and Carol's separately and independently. Because her responsibility is greater than that of either one of the two defendants, she would recover nothing from either.

Angela is a doctor. She is considering which of three possible courses of treatment to use on her patient Brianna. Each has a very small risk of harmful side-effects. Which of the following statements most accurately states Angela's duty to Brianna? (A) Angela owes Brianna a duty to choose the course of treatment that is most likely to be successful in Angela's considered medical opinion (B) Angela must describe all three courses of treatment to Brianna, tell Brianna of the risks involved with each, and let Brianna choose which course of treatment she wants (C) Angela need only tell Brianna about the risks posed by the alternative courses of treatment if a reasonable patient in Brianna's position would regard the risks as material (D) Angela need only tell Brianna about the risks posed by the alternative courses of treatment if a reasonable doctor in her position would do so

(c). See, e.g., Matthies v. Mastromonaco, 160 N.J. 26, 733 A.2d 456 (1999). Answers (a) and (d) do not give sufficient recognition to the idea of patient autonomy in the choice of courses of treatment; answer (b) gives too much recognition to that idea.

Andrew is a very tidy man. He is very annoyed by the city council's failure to clear the public ditches near his home. He repeatedly asks the city council to clear the ditches, without success. He decides to clear them himself, although he knows that he has a heart condition. While clearing the ditches, he suffers a heart attack. He sues the city council, which responds by arguing that Andrew was at fault himself, by doing hard labor when he knew he had a weak heart. In which of the following jurisdictions would it be impossible for Andrew's claim to fail altogether if the city council was held to be at fault? (A) A jurisdiction where contributory negligence is still a complete defense (B) A jurisdiction with a comparative fault regime that does not supersede the doctrine of intervening cause (C) A jurisdiction with a comparative fault regime that does supersede the doctrine of intervening cause (D) A jurisdiction that has abolished the rules of joint and several liability in favor of a system of several liability

(c). This question is simpler than it might seem at first. It would always be possible for a court to conclude that Andrew was the sole cause of his own loss unless the jurisdiction in question has abandoned the doctrine of intervening cause in relation to the plaintiff's own fault. Thus, it would be possible for Andrew's own fault to be regarded as the intervening cause in (a), (b) or (d).

Andrew is injured in an accident caused by the negligence of Bob and Clarence. Andrew suffers $100,000 worth of harm. Bob is 70% to blame, Clarence 30%. Clarence has no money and has fled the country. How much can Andrew recover from Bob in a state that has a system of several (not joint and several) liability? (A) $0 (B) $30,000 (C) $70,000 (D) $100,000

(c). Under a system of several liability, no person can be liable beyond his or her share of responsibility. The plaintiff bears the risk of insolvency on the part of potential defendants.

Implied assumption of risk is usually divided into two sub-categories: primary and secondary implied assumption of risk. Which of the following most accurately describes what "secondary implied assumption of risk" means? (A) The plaintiff voluntarily accepted secondary or incidental risks of the activity in question as well as its inherent risks (B) The plaintiff's assumption of risk defense is of secondary importance, so it will not operate in a system of apportionment of liability because of comparative fault (C) The plaintiff voluntarily assumed the secondary risk of not having primary risks explained to him or her by the defendant (D) The plaintiff was aware of and voluntarily accepted the risk of negligent conduct on the part of the defendant

(d). "Secondary" implied assumption of risk is so called to differentiate it from "primary" implied assumption of risk. In fact, there is nothing particularly "secondary"about either the risk in question of the plaintiff's assumption of it, so answers (a) through (c) are incorrect.

Which of the following statements most accurately states what the outcome should be when injuries caused by the defendant's negligence are made worse by subsequent negligent medical treatment? (A) The defendant should not be liable for the worsening of the plaintiff's condition because the defendant did not cause the additional harm (B) The defendant should not be liable for the worsening of the plaintiff's condition because negligent medical treatment is never reasonably foreseeable (C) The defendant should be held liable for the worsening of the plaintiff's condition because negligent medical treatment is always a possible consequence (D) The defendant should be held liable for the worsening of the plaintiff's condition only if the negligent medical treatment was reasonably foreseeable

(d). Answer (a) is wrong because it suggests that a defendant is always relieved of liability by a subsequent act of negligence, which is incorrect. Answer (b) overstates the case: surely some cases of negligent medical treatment are foreseeable. Answer (c) overstates the case in the opposite direction: surely it is not always true to say that negligent medical treatment is foreseeable.

Aaron is blind. He runs a concession stand in the lobby of the local post office. He usually walks with a cane while out in the street, but he does not use the cane while at work because he is so familiar with his surroundings. He goes to the rest room, leaving his cane behind. Aaron bumps into Bruce and knocks him over, injuring him. Which of the following statements is most accurate? (A) Aaron was negligent because a reasonably sighted person would not have bumped into Bruce (B) Aaron was negligent because a reasonable blind person would have used his cane at all times (C) Aaron was not negligent because it is not reasonable to expect him to have seen Bruce (D) Aaron was not negligent because it was not unreasonable for him to leave his cane behind given his familiarity with the surroundings.

(d). Answer (a) makes no allowance for physical disability, which would not be appropriate in these circumstances. Answer (c) makes allowance for physical disability but would excuse almost any conduct on Aaron's part. Answer (b) is closest to being correct; it at least applies the correct standard, which is that of the reasonable blind person. Answer (d) is most likely, however. That was the result in the case on which this question is based, Roberts v. Louisiana, 396 So. 2d (La.App 3 Cir. 1981).

Alan has a history of mental illness. He often suffers from severe paranoia. He has kept his condition under control for years by using medication. While he is out driving one day, he suddenly and unexpectedly forms the view that the driver of one of the cars on the other side of the road intends to kill him by crashing into him. (This is not true. The other driver has no such intention.) In order to avoid what he thinks is the impending collision, Alan drives his car onto the sidewalk, where he runs down Beltran, a pedestrian. Which of the following most accurately describes what will happen if Beltran sues Alan? (A) Alan's conduct will be judged according to the standard of the reasonable person with paranoid delusions (B) Alan's conduct will be judged according to the standard of the reasonable person under medication to control paranoid delusions (C) Because the attack of paranoia was unexpected, Alan's conduct will be judged according to what was reasonable in "the agony of the moment" (D) Alan's conduct will be judged according to the standard of the reasonable driver

(d). Mental illness is not a factor that would be taken into account in determining the standard of care to be taken in an activity like driving a car. Answers (a), (b) and (c) would all take Alan's paranoia into account to a greater or lesser extent, so all are incorrect.

Ann kisses her husband Brian goodbye as he prepares to go to work on his motorcycle. Ann steps back and is immediately horrified to see Brian knocked from his motorcycle by the negligent driving of Charles, who crashes his car into Brian's motorcycle. Danielle, a pedestrian passing by on the other side of the street, witnesses the accident, too. Brian suffers horrific injuries. Both Ann and Danielle suffer acute distress and recurring psychological problems as a result of witnessing the accident. What would the likely outcome be if Ann and Danielle sued Charles in a state that uses the traditional "impact" rule to determine liability in cases of negligent infliction of emotional harm? (a) Ann and Danielle would both win (b) Ann would win but Danielle would lose (c) Ann would lose but Danielle would win (d) Ann and Danielle would both lose

(d). Neither Ann nor Danielle suffered any physical impact themselves as a result of Charles's negligent driving so neither could succeed in recovering damages for negligently inflicted emotional harm under the traditional "impact" rule.

Anita is injured in an accident caused by the negligence of Barbara, Christine and Diana in a state that has not modified the rules of joint and several liability. She suffers $100,000 worth of harm. She sues Barbara, Christine and Diana. Anita settles with Barbara for $60,000, but proceeds to trial against Christine and Diana. The court determines that Barbara was 40% responsible, Christine 40% and Diana 20%. (In other words, Barbara settled for more than the court ultimately decided was her fair share.) What impact should the settlement of $60,000 already made by Barbara have on the liability of Christine and Diana under the "pro tanto" rule? (The "pro tanto" rule is preferred by the Restatement (Second) of Torts, Section: 886A(3).) (a) Anita should be able to recover $100,000, the full amount of the harm she suffered (b) Anita should be able to recover $66,667, because she is recovering from two of three jointly and severally liable defendants (c) Anita should be able to recover $60,000, the proportion of Anita's loss for which Christine and Diana were liable (d) Anita should be able to recover $40,000, the amount of the $100,000 loss that has not already been satisfied by Barbara

(d). The "pro tanto" rule gives the non-settling defendants a dollar-for-dollar credit for the amount paid by the settling defendant. Christine and Diana benefit from the fact that Barbara settled for more than her fair share, but Anita ends up getting no more and no less than her full loss. Remember, there is a separate question whether the defendants can claim contribution from one another under the pro tanto approach. The Restatement (Second) on Torts is silent about contribution: see the Caveat to § 886A(3).

Alberto's boat is damaged beyond repair when it runs aground while he is navigating using a chart published by Bonehead Charts, Inc. The chart is misleading. Alberto sues the publisher of the encyclopaedia in a state where the products liability law is modeled on Section: 402A of the Restatement (Second) on Torts. Which of the following most accurately states the likely outcome? (a) The publisher is not liable because the information in the chart is not a "product" for the purposes of strict liability under Section: 402A (b) The publisher is not liable under Section: 402A because the information in the chart can only harm someone who chooses to rely on it (c) The publisher is strictly liable under Section: 402A because the information in the chart was a defective product for the purposes of Section: 402A (d) The publisher is strictly liable under Section: 402A because the chart is a product that caused Alberto harm

(d). The answer to this question is different from the answer to Q 61. In the case on which Q 61 was based, the court said: "The Encyclopedia of Mushrooms is like a book on how to use a compass or an aeronautical chart. The chart itself is like a physical "product' while the "How to Use' book is pure thought and expression." Winter v. G.P. Putnam's Sons, 938 F.2d 1033, 1038 (9th Cir. 1991).

Andrew throws a baseball towards Barry, with whom he is playing pitch and catch. Andrew's aim is bad and the ball hits Cassandra, who is walking nearby. (A) Andrew has committed battery against Cassandra because his intention to hit Barry with the ball is transferred to Cassandra (B) Andrew has not committed battery against Cassandra because he did not intend to harm her (C) Andrew has committed battery against Cassandra because he intended to throw the ball, which hit her (D) Andrew has not committed battery against Cassandra because the contact he intended to make with Barry was not tortious because Barry consented to it

(d). The requisite intent to commit battery can be transferred, which is what answer (a) suggests, but the intent that is transferred must be the intent needed to commit battery. Answers (b) and (c) are based on the wrong kind of intent, regardless of the issue of transfer.

Andrew is injured in an accident caused by the negligence of Bob and Clarence. Andrew suffers $100,000 worth of harm. Bob is 70% to blame, Clarence 30%. Clarence has no money and has fled the country. How much can Andrew recover from Bob under the rules of joint and several liability? (A) $0 (B) $30,000 (C) $70,000 (D) $100,000

(d). Under a system of unmodified joint and several liability, each person causing harm is liable in full for the harm caused by all. Each defendant bears the risk of insolvency on the part of other potential defendants.

Sandler v. Commonwealth - RULE

- A plaintiff asserting a claim of recklessness against a state governmental agency for failure to act must show that the agency intentionally or unreasonably disregarded a risk that presented a high degree of probability that substantial harm could result. -

Peterson v. Taylor

- a seven year old boy should have acted with reasonable SOC of someone his age -he went into his neighbors backyard and lit himself on fire on accident

Cay v. State of Louisiana

- must show that defendant's actions were "but for" cause

Walmart Stores v. Wright

- we dont care about what the store said their standard for reasonable care was, we just care what the objective reasonable person would do as a standard of care

What is attractive nuisance?

-An exception to the usual treatment of trespassers -It states that a landowner may be held liable for injuries to children trespassing on the land if the injury is caused by an object on the land that is likely to attract children. -landowner subject to liability from children trespassing because of an artificial condition on the land IF: a: landowner has knowledge that it it likely kids will trespass b: landowner has knowledge of dangerous condition c: kids don't know/appreciate the risk d: it is not costly to mitigate the risk e: landowner fails to exercise reasonable care

Two types of emotional injury

-Emotional Harm Derivative of Physical Harm -plaintiff physically injured and p suffers emotional harm in process of being physically injured (fear, terror), OR as a result of physical injury going forward (pain and suffering, anxiety) -Pure Emotional Harm - plaintiff suffers no physical harm and is suing for pure emotional harm resulting from D's negligence; plaintiff is ALMOST physically injured and emotionally scarred by it (near miss); there was no threat of physical harm to plaintiff but they are emotionally scarred by the shocking accident; plaintiff is emotional harmed from witnessing somebody else be physically injured

Rabideau v City of Racine

-Observing serious injury to one's dog cannot be a basis for the type of bystander recovery that is sometimes available where the injury is suffered to a close relative that is a human -a cause of action for emotional distress suffered as a result of destruction of property is not recognized

Example of case involving duty owed to invitee

-Richardson v. Commodore - ceiling fell on man in bar - they should have known. -Duty for landowner operating place of entertainment higher than owner of private property.

in mesothelioma cases, how do we determine who caused the mesothelioma when there are multiple exposures from multiple defendants?

-The "But for" test does not work here because of the long latency period - we reject the substantial contributing factor test and instead just say that "if concurrent negligent acts are performed by more than one defendant, each defendant may be held liable

Ludman v Davenport Assumption High School

-boy hit by line drive in visiting team's dugout - A party may introduce evidence of custom to demonstrate that the party did not act negligently, particularly if there are no statutes defining proper behavior.

Wawanesa Mutual Insurance v. Matlock facts

-boys illegally bought cigarettes -one of the cigarettes accidentally burned down some wood - couldn't invoke negligence per se because the statute they violated (about underage smoking) was related to the damage that happened

Vaughan v. Menlove

-created reasonable person standard -Facts: Farmer ignored warnings that his hay was going to catch on fire. They caught on fire and burned down his neighbors cottages. Holding- A man must use and enjoy his property as to not injure that of another.

Ryals v US Steel (trespasser case)

-distinction between mere trespassers and criminal trespassers -some jurisdictions say that landowner only has to refrain from intentional torts ; others where trespassers are frequent and tolerated, landowners may be required to use reasonable care to protect them

Borders v. Roseberry:

-landlord under no obligation to repair/remedy defective condition on leased premises known to both landowner and tenant at time of lease, subject to 6 exceptions: 1. Undisclosed dangerous conditions known to lessor but not lessee (latent conditions) 2. Conditions dangerous to persons outside of premises 3. Premises leased for admission of public (storefront) 4. Parts of land retained in lessor's control that lessee entitled to use (common area) 5. When lessor contracts to repair 6. Negligence by lessor in making repairs.

Dover Elevator Co. v. Swann

-no need to invoke Res Ipsa Loquitur if there is sufficient evidence Sufficient direct evidence of negligence by the defendant will preclude application of the doctrine of res ipsa loquitur, which allows an inference that the defendant was negligent.

McQuiggan v. Boy Scouts of America

-the boy never expressly said he was DONE playing the game when he was hit; he consented to playing -A plaintiff who consents to participate in a sport, game, or contest in which injury is possible does not have a claim in tort if he sustains an injury as a result of his participation.

Woodard v. Turnipseed

-the kid who the farmer attacked with a broom after he wouldn't leave his property -A defendant claiming self-defense or defense of property to a charge of assault or battery must show that: (1) he used reasonable force not intended to cause death or serious bodily harm, (2) to thwart an unprivileged harmful or offensive contact or other bodily harm, (3) he believed was about to be imminently inflicted upon him.

Burke v. Schaffner

-the one where someone (Malone or Schaffner) hit the accelerator and pinned Burke against a car - alternative liability would have applied if both parties had been named but Malone wasn't named so alternative liability couldn't apply 1. Alternative Liability Theory applies if concurrent tortfeasors breach duties to P 2. Only 1 of whom harmed P & P can't tell which 3. Each created a similar risk of harm to P 4. All whose conduct might have injured P are being sued by P

Young v. Warren

-the one where the dad shot the boyfriend in the back as he left An individual in a civil action may assert a claim of defense of family to justify an assault on a third party, so long as the force used was not in excess of that necessary or reasonable.

Cullison v. Medley

-trailer park case -Assault requires that a person acts while intending a harmful or offensive contact with another or acts while intending to produce a reasonable apprehension of an imminent battery, causing mental or emotional damages. - assault was satisfied because the guy was shaking a gun in his face

Rule from Kingston v. Chicago & Northwestern Railway Co

-two fires, one from Railroad and one from unknown source - plaintiff recovered ALL damages from the railroad because the other party was identified -When two or more human entities both proximately cause injury to a plaintiff, and only one is identified, the plaintiff may recover the full amount of damages suffered from the one known wrongdoer.

What doctrine applies to product liability cases?

1. Applies to product liability cases and rescuer must still prove that tortfeasor's conduct was proximate cause of injuries.

6 factors that make fear reasonable:

1. Character and reputation of the attacker 2. The belligerence of the attacker 3. A large difference in size and strength btwn the parties 4. An overt act by the attacker 5. Threats of serious bodily harm 6. The impossibility of peaceful retreat

Custom evidence is inadmissible if

1. Custom does not extend to conduct at issue 2. Act is clearly careless or dangerous

reasons NIED is bad

1. Fear of Fabricated emotional harms 2. inability to measure emotional damages 3. Fear of excessive or unlimited liability (ex: 10000 witnesses to a plane crash) 4: skepticism about severity of Emotional Harms

Kingston v Chicago takeaway from Ellie's notes

1. If acts of 2 concurrent tortfeasors combine to produce a harm and either act would have been enough to produce harm, burden is on each tortfeasor to show its act was not a legal cause of harm by proving a. Other contributing cause was Not a tortious act, or b. Other's act created a force so much greater than tortfeasor's act that tortfeasor's act was not a proximate cause of harm

Points that Thomas v McDonald told us about negligence per se?

1. Plaintiff is a member of the class that the statute was meant to protect 2. Harm suffered is the kind of harm the statute was meant to avoid 3. Statute actually was violated

5 factors of substantial assistance

1. The nature of the wrongful act 2. The kind and amount of the assistance 3. The relation of the defendant and the actor 4. The presence or absence of the defendant at the occurrence of the wrongful act 5. The defendant's state of mind

6 exceptions that landlords ARE liable for:

1. Undisclosed dangerous conditions known to lessor but not lessee (latent conditions) 2. Conditions dangerous to persons outside of premises 3. Premises leased for admission of public (storefront) 4. Parts of land retained in lessor's control that lessee entitled to use (common area) 5. When lessor contracts to repair 6. Negligence by lessor in making repairs.

Ford Motor Co v Boomer

1. Where multiple acts occur, each of which would have been a but for cause of P's harm in absence of other acts, each is regarded as a factual cause of harm a. Substantial factor test plays no role in determining causation b. P must prove that each D's conduct alone was sufficient to have caused harm

a person acts with recklessness in engaging in conduct if:

1. the person knows of the risk of harm created by the conduct or knows facts that make the risk obvious to anyone in their situation AND 2. the precaution that would eliminate or reduce that risk involves burdens that are so slight relative to the magnitude of risk as to render the person's failure to adopt the precautions a demonstration of the person's indifference to the risk

Prima Facie elements of trespass

1: The defendant had the intent to invade the land 2: The defendant invaded the land 3: The plaintiff possessed the land and did not consent to the defendant's invasion If the plaintiff is not able to prove one of the components, then a court will likely find that the tort did not occur.

What is the reasonableness test for fear from Slayton v McDonald?

1: character/reputation of the attacker 2: belligerence of the attacker 3: large difference in size and strength between the parties 4: overt act by the attacker 5: threat of serious bodily harm 6: impossibility of peaceful retreat

Rescuer Status Test:

1: defendant was negligent to the person rescued and caused peril or appearance of to the person rescued 2: Peril/appearance of peril was imminent 3: Reasonably prudent person would conclude that peril/appearance of peril existed 4: Rescuer must act with reasonable care in effectuation of rescue

Three elements of trespass

1: if person enters land in possession of the other or causes a thing or a third person to do so OR 2: remains on land OR 3: fails to remove from land thing on which he is under duty to remove

what must you show to recover under the attractive nuisance doctrine?

1: there is a dangerous condition present on the land of which the owner is or should be aware 2: the owner knows or should know that young persons frequent the vicinity of this dangerous condition 3: the condition is likely to cause injury or is dangerous because of the Childs inability to appreciate the risk 4: the expense of remedying the situation is slight compared to the magnitude of the risk

What is the Dillon Test?

3 factor test: allows someone to recover even when outside the zone of danger 1: plaintiff must have proximity to scene of harm 2: shock must result from direct observation of accident 3: plaintiff must be closely related to victim (tenuous relationship = higher burden)

Arthur is injured in an accident caused by the negligence of Beau and Cat. A jury finds that Beau is 30% responsible for Arthur's injuries , Cat is 30% responsible for Arthur's injuries and Arthur is 40% responsible for Arthur's injuries. In a state that has a pure system of comparative fault what percentage of Arthur's loss can Arthur recover?

60%

Andrea is a doctor. While she is sitting having dinner with friends on the terrace of a restaurant, she sees Bill fall from his bicycle after riding over a pothole in the road. Bill is not wearing a helmet and his head is split open when it hits the ground. He lies there unconscious. Andrea does nothing to help him. She goes on eating dinner with her friends. Bill is eventually treated by paramedics who come to the scene, but he suffers permanent brain damage. The brain damage would have been avoided if he had been treated earlier. Which of the following statements is most accurate? (a) Andrea owed no legal duty to help Bill, who was not her patient (b) Andrea owed a legal duty to help Bill because her medical training makes her better able to help him than any of the other bystanders (c) Andrea owed a legal duty to help Bill because the Hippocratic oath taken by doctors requires them to swear to help others and liability for medical negligence is based on the standards of the medical profession, which creates a special relationship between Andrea and Bill (d) Andrea owed a legal duty to help Bill because she had a strong moral duty to do so and the law does not subvert morality

A - DOCTORS ONLY OWE DUTIES TO HELP PATIENTS BUT if a doctor invtervenes they then have assumed the legal duty to take reasonable care of the person and must do so to the best of their ability

which is likely to invoke res spa loquitur ? a 5 month old baby or a toddler coming home from day care with a broken arm?

A 5 month old baby - the 3 year old could've broken its arm in many other ways so that doesnt necessarily prove circumstances that he got hurt by negligence of the daycare workers

does battery always lead to an assault charge as well?

A battery may occur without assault if the victim does not perceive the impending contact

Poyner v. Loftus

A blind individual may be contributorily negligent in his actions if he fails to exercise due care for his own safety with respect to known or reasonably foreseeable dangers.

prima facie case

A case in which the plaintiff has produced sufficient evidence of his or her claim that the case will be decided for the plaintiff unless the defendant produces evidence to rebut it.

Byram v. Main

A defendant may be held liable for harm caused by a domestic animal with no history of being abnormally dangerous only if the owner intentionally caused the animal to do the harm or was otherwise negligent in failing to prevent the harm. Ray Byram (plaintiff) filed a negligence suit against Peter Main (defendant) after a pet donkey belonging to Main's daughter escaped from an enclosure, wandered onto a public interstate highway, and was struck by a tractor-trailer rig driven by Byram. After a trial, the trial court directed a verdict for Main on Byram's negligence claim. Byram appealed

Thomas v McDonals Rule

A defendant's violation of a state statute constitutes negligence per sewhen the plaintiff is injured as a result of defendant's conduct.

Zone of Danger Test

A duty of care is owed to an unforeseeable plaintiff, such as a bystander, when the plaintiff was within the foreseeable zone, or orbit, of danger. - EVEN IF no harm or impact if outside zone = no claim

Vaughan v menlove RULE

A person has a legal duty to use his or her property with the same level of ordinary care that would be exercised by a reasonable person.

McQuiggan v. Boy Scouts of America

A plaintiff who consents to participate in a sport, game, or contest in which injury is possible does not have a claim in tort if he sustains an injury as a result of his participation. actor may relinquish right to be free from harmful/offensive contact; consent and withdrawal of consent impliedfrom kid's manifestations of willingness to be contacted

Intentional Torts and Defenses - two requirements

Act Requirement/Intent Requirement

negligence per se

An action or failure to act in violation of a statutory requirement.

Concert of Action

An expanded liability concept that applies when all defendants acted together or cooperatively

alternative liability

An expanded liability concept that shifts the burden of proof to each of several defendants in a tort case when there is uncertainty regarding which defendant's action was the proximate cause of the harm. exposes an actor to liability even where there is a possibility that the plaintiff's harm was entirely caused by someone else

clark-aiken co v. Cromwell-wright Co

An individual or entity that carries on an abnormally dangerous activity is subject to liability for harm resulting from the activity even though the utmost care to prevent such harm was exercised.

Martin v. Herzog

An omission, or failure to perform an act required by statute, constitutes negligence per se.

Andy and barb are both 5 years old they are throwing stones trying to knock a tin can off a tree stump . Barb puts the can back on the stump after she has knocked it off. While Barb is standing close by the tree stump Andy throws a stone which hits Barb on the head. Which of the following statement most accurately describes the likely outcome if Barb sues Andy

Andys conduct will be judged by the standard of the reasonable 5 year old child because playing Is a child like activity

Andrea is a doctor. While she is sitting having dinner with friends on the terrace of a restaurant, she sees Bill fall from his bicycle after riding over a pothole in the road. Bill is not wearing a helmet and his head is split open when it hits the ground. He lies there unconscious. Andrea sees a group of bystanders gathering around Bill. She leaves the restaurant and says to the bystanders, "I'm a doctor but I can only help for a while. You should call an ambulance." She directs the bystanders to move Bill to the sidewalk and she stays with Bill for a while. She does nothing to treat him. She then returns to the restaurant to pay her bill and leaves. Bill is eventually treated by paramedics who come to the scene, but he suffers permanent brain damage. The brain damage would have been avoided if he had been treated earlier. Which of the following statements is most accurate? (a) Andrea owed no legal duty to help Bill, who was not her patient (b) Andrea owed no legal duty to Bill because she never touched him; the other bystanders did (c) Andrea assumed a legal duty to take reasonable care for Bill's safety once she intervened (d) Andrea owed a legal duty to Bill but she satisfied it by telling the bystanders to call for an ambulance

Answer (a) would be correct if Andrea had done nothing, but she must finish what she started. By taking some steps she assumed a duty to take reasonable care. Answer (d) is partly correct in that it says Andrea owed a duty, but reasonable care would probably require more of her in the circumstances. Answer (b) focuses on an irrelevant consideration, that Andrea never touched Bill.

Definition of Apprehension in terms of Assault?

Apprehension (R§27): defendant liable if intending or successful in putting plaintiff in apprehension of immediate bodily contact

Cullison v Medley

Assault requires that a person acts while intending a harmful or offensive contact with another or acts while intending to produce a reasonable apprehension of an imminent battery, causing mental or emotional damages

Doner has a heart attack while driving and Doners car hits Pentagone who is crossing the street in front of Doner. Which is correct? a. Doner is liable to Pentagone for battery b. Doner is not liable to Pentagone for battery because of involuntary conduct c. Doner is not liable to Pentagone for battery because Pentagone assumed the risk d. NONE OF THE ABOVE

B

John checks John's gun to make sure that it is not loaded before handing the gun over to Hans for Hans to examine. Hans thinking the gun is loaded points the fun at John and says "Ive always wanted you out of the way and nows my chance" as Hans pulls the trigger. Which statement is correct? a. this is assault b. this is not assault because John knows the gun is not loaded c. this is battery d. B and C e. none of the above

B

Learned Hand Formula

B<PxL. The defendant's burden to have acted differently, or the reasonable precaution cost, is balanced against the probability of harm(P), multiplied by the severity of harm(L). Defendant breaches their duty where the burden is less than the probability times the liability. -an actor is negligent if B is less than P

if someone kisses you on the cheek while you're asleep and they know you're mad at them - explain why this could be battery?

Because it doesn't matter that they were sleeping - if they had been awake they still would have been mad about it and found it offensive

Leichtman v WLJ Jacob Communications

Blowing smoke in another's face = battery; offensive contact = battery; offensiveness determined by offense of reasonable sense of dignity

Klein v. Pyrodyne Corporation

Brief: Plaintiffs were injured when an astray firework exploded near them. Defendant was sued under the tort of liability. Holding: trial court granted favor to plaintiff, defendant appealed to Washington Supreme Court. Strict Liability was appropriate Critical Facts: Rylands V. Fletcher established that defendant will be liable if the activity is unduly dangerous and inappropriate. Backed up by the restatement second of torts. The operator of a public fireworks display is strictly liable for harm resulting therefrom.

Which case explains Apprehension in terms of assault?

Brower v. Ackerley: threatened physical harm must be imminent for conduct = assault; words not enough unless also in combination with acts/circumstances causing fear/apprehension.

Multiple Sufficient Causes - Ellie's outline

But For test could prevent recovery in a case where conduct by each of the two or more actors was sufficient independently to have cause the plaintiff's harm **some courts allow defendants to show that its actions were not a substantial factor in causing the harm some courts require defendants to rebut the plaintiffs evidence by showing that its actions were not sufficient

Dillon v Legg

Bystander recovery allows n a cause of action for emotional distress caused by seeing person injured by defendant's negligence even if n outside ZOD if: 1: plaintiff is at scene 2: sees injury directly 3: has close relationship with victim

James v Lieb

Bystander recovery allows π cause of action for emotional distress caused by defendant's negligence even if π outside zone of danger if π: 1. Has significant attachment through marriage or intimate familial relationship with victim 2. Shock follows closely on heels of accident 3. Victim seriously injured/killed.

Grotts v Zahner

Bystander recovery requires n's relationship to victim be immediate family 1: other family members judges by FOF with regards to closeness

Legal Cause =

Cause in Fact

Landlord/Tenants

Common law immunity of landlords to tenants for safety of leased premises limited over time ii. Landlord/tenant relationship in and of itself insufficient to make landlord liable for tenants torts

tenants

Common law rejected tenant/guest classification as invitees Borders v. Roseberry the relationship of landlord/tenant is not in itself sufficient to make the landlord liable for tortious acts of the tenant When landlord leases to tenant, tenant becomes temporary owner/occupier, subject to responsibility No liability upon the landlord for defective conditions existing at the time of the lease EXCEPT Undisclosed dangerous conditions known to the lessor and unknown to the lessee Only latent conditions Conditions dangerous to persons outside the premises Premises leased for admission of the public (storefront?) Parts of land retained in lessor's control which lessee is entitled to use Where lessor contracts to repair Negligence by lessor in making repairs

three defenses to intentional torts

Consent Defense to self and others Defense of your land or personal property

when can implied consent be used to save someones life?

Consent to bodily contact can be implied as a matter of law if the contact is necessary to save the plaintiff's life. However, four elements must be present in order for the consent to be implied by law: The plaintiff is unconscious and unable to decide whether to grant or withhold consent. An immediate decision is required. There is no reason to believe that the plaintiff would withhold consent if he were conscious. A reasonable person in the plaintiff's position would consent.

Aaron is severely injured by Brandon's negligence. Aaron has been in a coma since the accident and remains so at the trial of his action against Brandon. Which of the following statements most accurately states what will happen at trial in relation to the award of damages for pain and suffering? (a) Aaron will recover no damages at all for pain and suffering because he has no subjective experience of pain or suffering (b) Aaron will recover substantial damages for pain and suffering to reflect the fact that he has been deprived of consciousness altogether (c) Aaron will be awarded pain and suffering damages that will be put into a trust fund and returned to Brandon if Aaron does not recover consciousness (d) The court will adjourn consideration of pain and suffering damages until it is clear whether Aaron will ever recover consciousness

Correct (a). Damages for pain and suffering are awarded on an entirely subjective basis, so (a) is the correct answer and (b), which depends on an objective assessment, is not. Answers (c) and (d) sound ingenious but they will not happen. The court makes an assessment of plaintiff's damages once and forever at the time of the trial.

Dar, while engaged in deep conversation with a passenger, takes Dar's eyes off road momentarily and hits pedestrian Palmer. Which is correct? a. Dar is liable to Palmer for battery b. Dar is not liable to Palmer for battery because of lack of intent c. Dar is liable to Palmer for negligence d. Both B and C above e. None of the above

D

Which of the questions would NOT be for a jury to decide? a. whether a defendant failed to take reasonable care b. whether reasonable care requires a driver to get out of his or her car to check for trains at an unmarked railroad crossing c. whether a traffic light was red when defendant drove through an intersection d. Whether a defendant owed a plaintiff a duty to take reasonable care

D

which of these is assault? a. shaking fist at P, Dagnar says, "Palome if you weren't a silly old man Id knock your brains out" b. "Im going to come to your house as soon as I can and beat you to a pulp said Dagon to Paris over the phone c. Ironside threatens to get out of his wheelchair and beat Pac silly and strains to get out of his wheelchair d. Darcy says to Patience "Im going to make jelly beans out of your brains and send them to your family" as Darcy gets up and moves towards Patience e. NONE of the above

D

Express consent (torts)

Defendant is not liable if plaintiff expressly consents to defendant's conduct.

Andrews v Peters

Defendant need not intend for plaintiff to suffer injury to be liable for battery; if contact is harmful/offensive, act is battery

Discounting methods - LOSS OF FUTURE EARNINGS

Discounting is the method used to reflect the present value of future earnings There are three approaches courts use to consider the impact of future inflation and productivity on lost future earning capacity.

Rescue Doctrine

Doctrine under which anyone who negligently causes harm to a person or property may be liable to one who is injured in an effort to rescue the imperiled person or property one who creates situation of peril has duty to individual who attempts a rescue in response to that situation in addition to the person imperiled.

What type of emotional harm/distress do all courts allow recovery from?

Emotional Harm derivative of physical harm

What is the impact test?

Emotional claims allowed if minimum physical impact occurs even when no injury --- fright does not suffice

Firefighter Rule

Firefighters/Police/Public Safety Personnel Cannot recover for injuries suffered caused by inherent risk of their job however if the property is unsafe the rule does not apply. 1. Cops/FF injured in LOD may not recover because of negligence requiring their presence.

For a design defect in a products liability action, the plaintiff usually must show that:A A less dangerous modification or alternative for the product was economically feasible.B The product emerged from production different from other products and more dangerous than if it had been made the way it should have been.C The product was dangerous because it departed from its intended design.D The product does not comply with government safety standards

For design defect cases, the plaintiff usually must show a reasonable alternative design, i.e., that a less dangerous modification or alternative for the product was economically feasible. Proof that the product emerged from production different from other products and more dangerous than if it had been made the way it should have been, or that the product was dangerous because of a departure from its intended design, establishes a manufacturing defect rather than a design defect. It is not necessary to show that the product does not comply with government safety standards to establish a design defect. A product may comply with the standards and still be found defective.

Shull v. B.F. Goodrich Co.

Gave us the two elements of RIL The doctrine of res ipsa loquitur is applicable if: (1) the injuring instrumentality was under the defendant's exclusive control, and (2) the accident is one that in the ordinary course of things does not happen if the party in control of the instrumentality uses proper care.

when do you have a duty to rescue

Generally, no duty exists — even between friends — unless the defendant either causes the peril or actually commences a rescue attempt (in which case she is obligated to use reasonable care in finishing the rescue)

What is harmful contact according to the restatement 84?

Harmful Contact (R§4): Physical Harm = physical impairment of human body or real/tangible property.

ely v cabot Oil and gas corp

Hydraulic fracturing is not an abnormally dangerous activity and thus is not subject to strict liability.

Transfer of Intent between Parties

If defendant intends to assault/batter one party but ends up w/ another defendant they are liable to the 2nd party as if it was the 1st.

Multiple Sufficient Causes

If multiple acts occur, each of which under §26 alone would have been a factual cause of the physical harm in the absence of the other acts, each is regarded as a factual cause of the harm. Illustration: two people camping independently in same forest negligently do not put out fires after going to bed. Both fires spread, join together, burn down company's lodge. Each of them is a factual cause of the destruction of the lodge. NOTE: "but for" test doesn't work here. See teratogenic example on P. 127.

Clark v Brings - animal injuries

In Minnesota, owners of cats may not be held strictly liable for injuries caused to individuals

Nelson v. Freeland

In North Carolina, determining premises liability based on an entrant's status as either an invitee or licensee is abandoned in favor of a true negligence standard which examines whether a landowner acted as a reasonable person would have under the same circumstances.

indivisible harm

Indivisible injuries can be injuries that cannot be separated, such as aggravation or exacerbation of an earlier injury; can be an injury to the same area of the body; or, can be global symptoms that are impossible to separate.

Battery - Two Tests?

Intent test Offensiveness Test

definition of assault

Intentionally causing reasonable apprehension of imminent harmful/offensive contactIntent requirement.

Joint and several liability

Joint and several liability is a rule followed in some states, in which two or more parties can be held independently liable for the full amount of a personal injury plaintiff's damages, regardless of their respective degrees of fault.

if I run at someone and they fear that I am going to hit them - and then they jump out of the way and sprain their ankle - and then they go to the hospital and the hospital catches fire and the person gets burned - what am I liable for?

Just the sprained ankle... the burns from the hospital were so unforeseeable/ were so much greater than the scope of the ankle sprain that it wouldn't be fair to hold me liable for them

What duty as a landowner do I have to licensees?

Landowner has duty to warn of non-obvious, hidden dangers knownto landowner

What duty as a landowner do I have to invitees?

Landowner has duty to warn/repair any unreasonable dangers and to discover dangers

What duty as a landowner do I have to trespassers?

Landowners only duty to refrain from willful/wanton injury (exception: of attractive nuisance)

Vicarious Liability

Legal doctrine under which a party can be held liable for the wrongful actions of another party. (children) i. Courts generally exercise vicarious liability for parents over child's torts if parents themselves at fault; some states hold vicariously liable even when parent's weren't at fault.

example of when failing to comply with a statute can be used as a defense?

Martin v Herzog - negligence per se; the plaintiff was driving without lights and they were supposed to have lights on

explain the STD question

Mistaken consent cannot authorize contact that would otherwise constitute battery.

Does a future threat (even for tomorrow) count as an imminent contact in terms of assault?

NO

can intentional conduct be negligent?

NO

do you have to intend to be on someones property to be liable for trespass?

NO

does the landowner have to know about the trespass for it to be trespass?

NO

intent - does the injury done has to be the one that's intended?

NO

is the ability to help a reason to have to do so?

NO (ex: doctors only have to help if the person is their patient) BUT if a doctor invtervenes they then have assumed the legal duty to take reasonable care of the person and must do so to the best of their ability

Do children have to conform to a higher standard of care when trespassing?

NO - but the adult on whose property they are on normally must if there is an attractive nuisance (attractive nuisance)

does battery have to be harmful?

NO - could also be offensive

do experts have duty to rescue?

NO - even if you are an expert, it does not create a duty to rescue

does there have to be harm to be able to recover for trespass?

NO - everyone just has a right to not have their property infringed upon

Is there a special duty to rescue in tort law?

NO - no general duty to rescue even when it is foreseeable

can you recover economic damages if an accident occurs next to your store that deters visitors?

NO - not unless your store was physically damaged

must a child act with a higher standard of care when engaging in a dangerous activity?

NO - only when engaging in an activity that is normally only for adults to engage in

does an injury doesn't have to be likely or probable in order to be foreseeable in a proximate cause analysis?

NO - throwing flower pot out window example

In terms of the Scope of Risk/Foreseeabliity test: would we hold a defendant liable for leaving their keys in a car that a terrorist then took to bomb a building?

NO because the defendant could not have foreseen that leaving the keys negligently in the car would have lead to it being stolen by a terrorist and a building being bombed

What if I see something about to harm another person and say nothing - can I be held liable?

NO unless you were the one who actually planted the thing that was going to cause the harm

Robinson v. Lindsay

Negligence--Duty Case: Snowmobile accident causes girl to lose use of her thumb (child vs. adult SOC) -we hold children to adult SOCs when doing adult activities

does battery have to result in harm?

No - just any actual contact that is "offensive"

Offensiveness test: for battery

Objective - general societal consensus, not what defendant knew or desired

Licensee definition

On landowner's land - owner receiving no financial benefit (social guest using vacation home)

Byrne v. Boadle

Origin of res ipsa loquitur; when an event occurs that would not normally occur without negligence and the defendant is in exclusive control of the instrumentality of harm, there is a rebuttable presumption of negligence against the defendant

Invitee definition

Owner receiving direct/indirect mutual benefit/economic advantage by plaintiff being on land (business customer)

which case gives us the majority approach to duty questions?

PALSGRAF - a plaintiff can recover for the defendant's breach of duty only if she can establish that a reasonable person would have foreseen a risk of injury to her in the circumstances; i.e., that she was located in a foreseeable zone of danger.

Pure comparative negligence v. Modified comparative negligence

PURE: plaintiff only barred from recovery if they are 100% at fault, can even recover for 99% (damages would be 1%) MODIFIED: type1 - negligent plaintiff can only recover if his negligence is less than or equal to 49% type2 - negligence plaintiff can only recover if his negligence is less than or equal to 50%

Harmful Contact (R4)

Physical harm = physical impairment of human body or real/tangible property

Shugar v. Guill

Recovery of punitive damages in an action for assault and battery is permissible upon a sufficient showing that the defendant's actions were accompanied by an element of aggravation such as malice, oppression, or gross and willful wrong, or a wanton and reckless disregard of the plaintiff's rights.

the two ways to prove breach?

Res Ipsa Loquitur Negligence Per Se

Battery definition

Right to be free from intentionally inflicted harmful/offensive contact

Intent Test: for battery

Subjective - defendant only has to intend contact to satisfy intent requirement

Cause in fact or but for causation cannot be apportioned. In other words, one either IS or IT NOT a but for cause - T/F

T

Open & Obvious Rule

The Open and Obvious doctrine is a defense used mostly in Premise liability cases. The general rule is that a premises possessor owes a duty to an invitee to exercise reasonable care to protect the invitee from any unreasonable risk of harm caused by a dangerous condition on the land.

A small cruise ship struck a whale swimming underwater, causing the ship to suddenly lurch sideways. A passenger on the ship who was walking down a corridor lost his balance and bumped his head on the edge of a doorway. Because of a previously existing medical condition that made him susceptible to bleeding on the brain, he suffered a cerebral hemorrhage and permanent mental impairment, despite prompt medical attention on the ship.The passenger brought suit against the cruise ship owner for his damages. At trial, the passenger presented evidence of how he was injured as he walked down the hallway, his previous medical condition, and his medical expenses and other damages. The cruise ship owner presented evidence that the cruise ship was following its approved route and that the whale could not have been detected before impact, and that the bump would not have injured someone in ordinary health. At the close of the evidence, the cruise ship owner moved for a directed verdict.How should the court rule?A Grant the motion, because there is no evidence that the crew operated the ship negligently.B Grant the motion, because the cruise ship owner introduced uncontroverted evidence that a person in normal health would not have been injured by the bump.C Deny the motion, because the jury could find that the cruise ship owner, as a common carrier and innkeeper, breached its high duty of care to the passenger.D Deny the motion, because the fact that the severity of the passenger's injuries was notforeseeable does not cut off the cruise ship owner's liability.

The court should grant the cruise ship owner's motion because the passenger has not established a prima facie case of negligence against the cruise ship. To establish a prima facie case for negligence, a plaintiff must show (i) a duty of care, (ii) breach of that duty, (iii) actual and proximate cause, and (iv) damages. As a common carrier and/or an innkeeper, the cruise ship owed its passengers a high duty of care, and therefore would be liable for slight negligence. However, the passenger has offered no evidence to establish that the cruise ship employees breached that duty, and res ipsa loquitur is not applicable here because the collision with the whale swimming underwater is not the type of event that would occur only as a result of negligence. Because the passenger failed to establish breach of duty, the court should grant the cruise ship owner a directed verdict. (B) is incorrect because the cruise ship owner does not need that evidence to prevail. While evidence that a person in normal health would not have been injured by the bump supports the cruise ship's other evidence that it exercised due care, it is not necessary because the passenger has failed to offer evidence that the cruise ship owner breached its duty. On the other hand, if the cruise ship owner had breached its duty of care to its passengers, the fact that a person in normal health would not have been injured by the bump on the head would not be a defense to liability. If a defendant's negligence causes an aggravation of a plaintiff's existing physical illness, the defendant is liable for the damages caused by the aggravation. (C) is incorrect because, as discussed above, the passenger has failed to present evidence that the cruise ship owner breached the high duty of care that it owed to its guests. (D) is incorrect even though it is a true statement of law, as discussed above. The reason the cruise ship owner prevails is because the passenger has failed to establish a prima facie case

Sudden Emergency Doctrine

The degree of care required of a person in an emergency situation is less than would be required if the person had time to think before acting. --- seen in Lyons

Foster v. Costco

The distraction exception to the open-and-obvious rule imposes liability upon a landowner for failing to remedy the risk of harm from an obvious hazard when a business invitee could be distracted from observing or avoiding the dangerous condition and the landowner has reason to expect that the invitee may suffer physical harm as a result.

A motorcyclist was injured in a collision and suffered $100,000 worth of injuries, including $20,000 in hospital and physician's bills. The motorcyclist's medical insurance company paid her $20,000 to cover hospital and medical expenses. Later, she filed suit against the driver of the car that struck her motorcycle. When the case came to trial, the jury agreed with the motorcyclist's contention that her injuries were worth $100,000. The jury also determined that the motorcyclist was 30% negligent and that the driver was 70% negligent.How much should the motorcyclist recover from the driver?

The motorcyclist should recover $70,000 from the driver. Under a comparative negligence system, a contributorily negligent plaintiff is allowed to recover a percentage of her damages. The plaintiff's damages are reduced according to her proportionate share of the fault. Thus, the motorcyclist can recover 70% of her total of $100,000 in damages because she was 30% at fault, leaving her with a recovery of $70,000. As a general rule, damages are not reduced or mitigated by reason of benefits received by the plaintiff from other sources, such as health insurance. Therefore, the $20,000 paid by the motorcyclist's insurance company will not reduce the $70,000 in damages to which she is entitled. (A) is incorrect because it fails to reflect the reduction in damages required under comparative negligence. Because the motorcyclist was 30% negligent, she cannot recover the entire $100,000. (C) is incorrect because it is derived from an initial reduction of damages by the amount of the insurance payments ($100,000 minus $20,000, leaving $80,000). This $80,000 figure is then reduced by the 30% negligence of the motorcyclist, leaving an amount of $56,000. However, as noted above, the insurance payments are not allowed to reduce damages. Thus, the 30% reduction is made from the figure of $100,000, not from $80,000. Similarly, (D) is incorrect because it is derived from a reduction of the $70,000 proportionate recovery by the $20,000 insurance payment.

A landowner had a swimming pool and a dressing cabana constructed in her spacious backyard. The pool was entirely within the confines of the landowner's property. However, one corner of the cabana extended a few inches onto a far corner of her neighbor's land. At the time of the construction, neither the neighbor nor the landowner was aware that the cabana extended onto the neighbor's property.Does the neighbor have a cause of action for trespass?A Yes, because the cabana extends onto the neighbor's land.B Yes, because the presence of the cabana on the neighbor's land has caused damage to his property.C No, because the landowner did not actually enter the neighbor's property.D No, because the landowner did not intend to have the cabana encroach on the neighbor's property.

The neighbor will prevail because the cabana extends onto the neighbor's land. The tort of trespass to land requires: (i) an act of physical invasion of the plaintiff's real property by the defendant, (ii) intent by the defendant to bring about a physical invasion of the property, and (iii) causation. The intent required is the intent to enter on a particular piece of land, rather than intent to trespass. Also, it is not necessary that the defendant personally enter the land. It is sufficient if the defendant's act or something set in motion thereby causes a physical invasion of the property. By having the cabana constructed, the landowner acted so as to bring about the physical invasion of the neighbor's land. (C) is incorrect because it makes no difference that the landowner herself did not enter the property that was being violated. (D) is incorrect because the landowner's intent to have the cabana built on its current site suffices for purposes of trespass liability. As noted above, the defendant need not have intended to commit a trespass. (B) is incorrect because actual injury to the violated property is not a prerequisite to sustain this cause of action. Damage is presumed.

There are three approaches courts use to consider the impact of future inflation and productivity on lost future earning capacity.

The traditional approach ignores the effects of each as being entirely too speculative. The middle ground approach permits the factfinder to consider the effects of inflation and productivity on lost future earning capacity, but prohibits expert testimony to be provided on either of the issues. Finally, the evidentiary approach permits consideration of each issue

Slayton v. McDonald

The use of a dangerous weapon to repel an attack is justified if an individual is in fear of an imminent threat of serious bodily harm or death.

Slayton v. McDonald

The use of a dangerous weapon to repel an attack is justified if an individual is in fear of an imminent threat of serious bodily harm or death. -the one where the kid dared the guy to shoot him and wouldn't leave and he was scared for his life so he shot him in the knee

Creasy v. Rusk

There are five main policy reasons for holding a person with mental disabilities to the same standard of care as a person of sound mind.

does a battery have to happen instantaneously?

Time-delay: no doesn't have to happen instantaneously ex: poisoning someone's food

What is a Childs normal standard of care

To act the same as a child of like age, education, intelligence, and experience

(cause-in-fact)

To win damages, the plaintiff must show a causal relationship between the defendant's conduct and the plaintiff's harm Requires proof as a matter of historical and physical fact, it is more likely than not that the defendant's conduct was a cause of what happened to plaintiff

what happens if you're trying to throw a brick at someone as self defense and it accidentally hits someone else?

Transfer of intent not applicable because you can't be held liable due to the fact that another persons actions gave rise to you having the right to throw a brick at them in the first place

Summers v. Tice

Under Summers v. Tice, if there is clear evidence that two or more parties were negligent, but only one could have caused an injury, and if it cannot be shown which of the negligent parties caused the injury, then the burden of proof is on each defendant to show that he did not cause the injury. Absent such evidence, the negligent parties may be held jointly and severally liable for the damages.

Wawanesa Mutual Insurance Co. v. Matlock

Unless legislation was intended to prevent the type of harm that occurred, proof of statutory violation has no relevance in a tort case

Tort of Nuisance

Unreasonable interference of another's rights to use and enjoy their property

Shinn v Allen

When a plaintiff wants to impose liability on a person who "assisted" the defendant who actually did the harm, they have to prove that he "substantially assisted" through a 5 factor test

When does a child need to conform to. higher standard of care than usual?

When doing an activity that normally only adults do

When does the attractive nuisance doctrine not apply?

Where the kid knows of danger, understands risk and is in position to avoid it, and voluntarily proceeds and is injured anyway

Does a defense have to be reasonable force?

YES

after you establish a but for cause, must you also then prove foreseeability?

YES

can you be liable for trespass if you leave something on someones property after the date you said you wouldn't?

YES

if you ignore trespass over time (for example a pipe on your property) can your right to recover go away?

YES

is permission to be on someones land a defense to trespass?

YES

intent for trespass is same as liability for assault/battery?

YES (belief that you are on property you're entitled to be on does not save you from trespass accusations)

can you still be charged with negligence even if you cant be charged under the negligence per se statute?

YES (ex: school bus case from E&E)

can a big bodybuilder guy hold a small little old lady liable for assault even if a reasonable person wouldn't think that he would be threatened by her words/actions due to the size disparity?

YES - can still hold her liable - because we don't care about the person's ability to defend themselves we just care about the fact that the little old lady's actions/words DID create an apprehension of unwanted contact, even if she couldn't really create substantial harm for the bodybuilder

if someone intends a battery - even if the party doesn't "apprehend" the fear of the contact at the time its happening - can it still be considered assault?

YES - could be liable. An attempted battery is an assault

does "consent" travel with transfer of intent between torts/parties?

YES - like playing catch and the ball hits someone else is not a battery

Is unexcused violation of a statute in itself negligence?

YES - negligence per se

would a conductor be liable for knocking a package down while trying to help someone enter the train and breaking the contents of the package even if they didn't know the contents of the package were an antique jewelry box worth $5000?

YES -- courts generallyy hold that people are liable if the GENERAL TYPE of foreseeable action occurred (the thing falling) EVEN IF the ACTUAL damage was much greater (like the fact it cost $5000) which was unknown to the conductor at the time

can someone who does not physically trespass themselves be liable for trespass?

YES if they caused the other person to trespass and there was intent

is a tortfeasor liable to a victim even if mistaken of their identity?

YES obviously - just be careful on these problems because they're not "transferred intent" they're just "mistaken identity"

in terms of proximate cause, is a defendant liable to a plaintiff who only had small lacerations but that ended up bleeding out because they were a hemophiliac?

YES they are liable to the whole thing - it doesnt matter than on a normal person the small lacerations wouldn't have been a big deal

if someone is threatening you with deadly force can you use deadly force back?

Yes

if there is a threat of imminent contact that DOES constitute assault, but the person who the threat is aimed at is super sensitive and then has to go to therapy for a million years after - is the person who was originally held liable for assault also liable for those damages (aka: are they liable for the later costs of therapy, etc)?

Yes

can children have affirmative duty to act?

Yes - but they are only required to conform to a standard of care that that of a child of similar age, education, iltelligence, and experience would

can children be held liable for assault?

Yes if the intent was to bring about the offensive or harmful contact

if you cant be charged under negligence per se can you still be charged for negligence?

Yes under the reasonable persons standard

when thinking of whether a "reasonable person" would find a contact offensive - keep in mind which example from the E&E?

Yes, liable for battery - hugging a girl on the first day of school that you don't know (she then found it offensive) No, not liable for battery - slapping a guy as congratulations in the course of a football game

prima facie case

a cause of action or defense that is sufficiently established by a party's evidence to justify a verdict in his or her favor, provided such evidence is not rebutted by the other party.

Polmatier v Russ

a psycho person can be held liable for tort resulting from battery/killing if actions that caused that harm were voluntary and likely to result in negative consequences - Acts not reflexive, convulsive, or epileptic; rational choice not required since insane person may have intent to invade interest of another even though reasons are irrational

Andrea is a doctor. She discovers that one of her patients, Barry, has cytomegalovirus (CMV), a mild and widespread virus that can be spread through sexual contact. Its symptoms are usually very mild but it can cause birth defects if contracted by a pregnant woman. Andrea knows that Barry is sexually active but he does not know that he has a relationship with Claire, who is pregnant. Claire contracts CMV after having sex with Barry, and she gives birth to a son, David, who has birth defects. Which of the following statements most accurately states the likely outcome if David sues Andrea? (a) Andrea owed no duty to inquire about David's sexual partners and no duty to warn any of them of the risks of CMV (b) Andrea owed a duty to all of David's sexual partners to warn them of the risks of CMV (c) Andrea owed Barry a duty to warn him about the risks of CMV (d) Andrea owed Clare a duty to warn her that Barry might have CMV

a). Answer (b) might seem plausible, but the relationship between Andrea and Barry is not such as to give rise to a duty to control. Although (c) is correct it is irrelevant in an action by David against Andrea. Answer (d) is both incorrect and irrelevant.

Ayrton, a racing-car driver with extraordinarily fast reflexes and superb car-handling skills, is driving along an ordinary suburban street in an ordinary, unmodified car. A child runs out onto the road so close in front of his car that an ordinary driver would have been unable to avoid running her down. Because of his extraordinary reflexes and skills, Ayrton could have avoided the child if he had been paying attention. However, he runs the child down because his attention is distracted by an attractive female pedestrian on the sidewalk and he is talking on his hand-held cell phone. Which of the following statements is most accurate? (A) Ayrton was negligent because a reasonable driver with his reflexes and skills would have avoided the child (B) Ayrton was negligent because he could have avoided the child but did not (C) Ayrton was not negligent because a reasonable driver in his position would not have been able to avoid the child (D) Ayrton was not negligent because someone with his extraordinary reflexes and skills is entitled to drive without paying full attention

a). Answer (b) suggests a subjective standard, which is inappropriate. Answer (d) seems inconsistent with any notion of reasonable care. The most difficult choice is between answers (a) and (c). Answer (c) is attractive because it holds Ayrton to the same standard as all other drivers, which is the usual goal of the supposedly objective reasonable person standard. Nevertheless, although a driver's subjective inability to drive (through age, health, etc) would not be taken into account when setting the standard of reasonable care, extra skill and ability usually is taken into account. Answer (a) accurately states the relevant test, which is what a reasonable driver with Ayrton's extra ability would have done.

Annabelle's dog Boris runs across the street and digs up the lawn of Catherine's house. Which of the following answers is most accurate? (a) Annabelle is liable to Catherine because she failed to take reasonable care to keep Boris from trespassing on Catherine's property (b) Annabelle is strictly liable for the harm done by Boris (c) Annabelle is not liable because it was Boris, not Annabelle, who did the damage (d) Annabelle is not liable because there is no vicarious liability for torts committed by animals

a). Answer (c) is incorrect because it would lead to the result that owners would never be responsible for damage caused by their animals. Answer (d) is incorrect because liability for damage caused by animals is not based on vicarious liability. Answer (b) is incorrect; there is no strict liability for damage done by household animals.

One of the factors to be taken into account in determining whether an award of punitive damages is unreasonable or excessive is the ratio of the punitive damages award to actual harm inflicted on the plaintiff, measured by the size of the compensatory damages award. There is no simple or rigid mathematical formula, but which of the following ratios most accurately states what would be regarded as acceptable before the award of punitive damages becomes constitutionally improper? (a) Not more than 4 to 1 (b) Not more than 25 to 1 (c) Not more than 100 to 1 (d) Not more than 500 to 1

a). B.M.W. of North America, Inc. v. Gore, 517 U.S. 559 (1996), Pacific Mutual Life Ins. Co. v. Haslip, 499 U.S. 1 (1991). In TXO Production Corp. v. Alliance Resources Corp., 509 U.S. 443 (1993), the Court upheld an award with a ratio of 10 to 1. That would still be considerably less than answers (b) through (d) and much close to (a).

Knorpp v Hale

a. A land entrant is an invitee if invited to enter or remain on land b. As a member of public for a purpose for which land is held open to public (public invitee), Or c. For a purpose connected with business dealings with possessor of land (business visitor) d. A land entrant is a licensee if invited in any other way or for any other purposes e. Social guests are usually licensees (footnote 3) f. Duty owed to invitee is to exercise ordinary care with respect to risks land possessor knows or should know with reasonable inspection g. Duty to licensee is ordinary care to warn about or make safe a danger that possessor knows and licensee does not know

Recklessness

a. All unintentional torts other than negligence including behavior described as "wanton misconduct" and "gross negligence" i. Can recover for punitive damages in some cases

Merrill v. Central Maine Power Co.

a. Attractive nuisance protects some child trespassers from application of ordinary limited duty rules b. Child's lack of knowledge of risk is a crucial element c. Restatement (2d) identifies 5 factors (a) - (e) to apply doctrine d. Note 2 Question

What kinds of people have duty to rescue and provide medical care in times of emergency?

a. Business proprietor, special relationships, & common carriers (plane, train, bus, etc. have duty to take reasonable action to provide appropriate medical care to patrons in times of emergency but not all possible medical care

Res Ipsa Loquitur

a. Circumstantial evidence is information a factfinder may use to make inferences about past events b. Some use of circumstantial evidence is used as evidence of breach (res ipsa loquitor)

Traditional Strict Liability

a. For injuries caused by some kinds of activities, tort law imposes liability without regards to the actor's fault. b. The plaintiff does not have to show that the defendant was at fault or engaged in tortious activity, or intended the injury, or acted negligently

Ryals v United State Steel Corp

a. In most jurisdictions, duty that a landowner owes to a trespasser is to refrain only from intentional or wanton injurious conduct b. Alabama recognizes this doctrine & elaborates it to create a sub-category of trespassers (namely those who enter land to commit a crime) to whom a lesser duty is owed (namely to refrain from intentional injurious conduct)

what are three scenarios when the "but for" burden shifts to defendants?

a. Multiple sufficient causes - concurrent tortfeasors, each sufficient to cause harm (the two fires) b. Alternative liability - concurrent tortfeasors, plaintiff cannot determine which caused harm (bird hunting example) c. Indivisible harms - concurrent tortfeasors, each of whom contributed inseparably to harm (multiple attackers kicking a guy in the head)

Concerted Action theory summarized in Restatement of Torts

a. Pursuit of a Common Plan, or b. Substantial Assistance Knowing of Breach of Duty, or c. Substantial Assistance and Actor also Breaches a Duty

Hill v National Grid

a. Restmt. § 339 imposes liability on a possessor only when possessor knew or had reason to knowabout dangerous condition & that children were likelyto trespass b. Footnote 5: "Reason to know" means D has information from which a person of reasonable intelligence would infer fact in question exists c. This rule differs from negligence, which requires D "should have known" of risk

what is the defense of consent?

actor may give up interest in freedom from harmful/offensive contact and apprehension thereof

Definition: Defense to self and others

actor may use force proportionate to interest protecting, injury/harm threatened by other party, the case law

if a plaintiffs injury is TRULY Beyond the type of harm to be expected from the defendant's conduct, will the plaintiff be compensated?

almost always NO

what 5 things does transferred intent apply to

assault battery false imprisonment trespass to chattels trespass to land

Ann kisses her husband Brian goodbye as he prepares to go to work on his motorcycle. Ann steps back and is immediately horrified to see Brian knocked from his motorcycle by the negligent driving of Charles, who crashes his car into Brian's motorcycle. Danielle, a pedestrian passing by on the other side of the street, witnesses the accident, too. Brian suffers horrific injuries. Both Ann and Danielle suffer acute distress and recurring psychological problems as a result of witnessing the accident. What would the likely outcome be if Ann and Danielle sued Charles in a state that uses the "zone of danger" rule to determine liability in cases of negligent infliction of emotional harm? (a) Ann and Danielle would both win (b) Ann would win but Danielle would lose (c) Ann would lose but Danielle would win (d) Ann and Danielle would both lose

b). Ann was in the "zone of danger" caused by Charles's negligent driving, as she could have been hit herself. Danielle was not in the "zone of danger" so she cannot recover.

Annabelle's sheep Boris strays from Annabelle's paddock and eats the prize flowers in Catherine's yard. Which of the following answers is most accurate? (a) Annabelle is liable to Catherine because she failed to take reasonable care to keep Boris from trespassing on Catherine's property (b) Annabelle is strictly liable for the harm done by Boris (c) Annabelle is not liable because she is only liable for the kind of harm typically done by sheep (d) Annabelle is not liable because there is immunity from harm done by farm animals

b). Answer (a) would be correct for household animals, answer (b) is correct for farm animals. Answers (c) raises irrelevant issues; answer (d) is the opposite of the correct answer.

Andrew and Barbara are 5 years old. They are throwing stones, trying to knock a tin can off a tree stump. Barbara puts the can back on the stump after she has knocked it off. While Barbara is still standing close by the tree stump, Andrew throws a stone, which hits Barbara on the head. Which of the following most accurately describes the likely outcome if Barbara sues Andrew? (A) Andrew's conduct will be judged by the standard of the reasonable adult because throwing stones is a dangerous activity (B) Andrew's conduct will be judged by the standard of the reasonable 5-year-old child because playing is a child-like activity (C) Barbara's action will fail because a reasonable child of her age would have realized that it was dangerous to stand next to the tree stump (D) Andrew's conduct will be judged by the standard of the reasonable minor, not taking into account his actual age because the test is objective, not subjective

b). Answer (c) is inappropriate because it suggests that fault on Barbara's part (even if established) is a complete defense, which is only true in very few states. Answer (d) is seductive, but incorrect: the test is objective, but it would take into account Andrew's age. Answer (a) gives insufficient weight to the context in which the accident occurred

Anita is injured in an accident caused by the negligence of Barbara, Christine and Diana in a state that has not modified the rules of joint and several liability. She suffers $100,000 worth of harm. She sues Barbara, Christine and Diana. Anita settles with Barbara for $20,000, but proceeds to trial against Christine and Diana. The court determines that Barbara was 40% responsible, Christine 40% and Diana 20%. (In other words, Barbara settled for less than the court ultimately decided was her fair share.) What impact should the settlement of $20,000 already made by Barbara have on the liability of Christine and Diana under the "pro tanto" rule? (The "pro tanto" rule is preferred by the Restatement (Second) of Torts, Section: 886A(3).) (a) Anita should be able to recover $100,000, the full amount of the harm she suffered (b) Anita should be able to recover $80,000, the amount of the $100,000 loss that has not already been satisfied by Barbara (c) Anita should be able to recover $66,667, because she is recovering from two of three jointly and severally liable defendants (d) Anita should be able to recover $60,000, the proportion of Anita's loss for which Christine and Diana were liable

b). The "pro tanto" rule gives the non-settling defendants a dollar-for-dollar credit for the amount paid by the settling defendant. The Restatement (Second) on Torts is silent about whether Christine and Diana should then be entitled to recover a contribution from Barbara. In a Caveat to § 886A(3), the American Law Institute preferred to say nothing at all about whether non-settling defendants should be able to get a contribution from settling defendants.

Question 2 Alice is injured by Brent's negligence. She receives $10,000 under a first party insurance policy (e.g., an accident insurance or health insurance policy). Which of the following most accurately states what will happen if she sues Brent? (a) The amount of damages she recovers will be reduced by $10,000 to take account of the fact that she has already been compensated for that much of her loss (b) The amount of damages she recovers will not be affected by the fact that she has received $10,000 from her insurer (c) The amount of damages she recovers will be increased by $10,000 so that she can repay her insurer without being left out of pocket (d) She can only recover damages from Brent if she pays back the $10,000 to her insurer

b). The collateral benefit or collateral source rule says that most benefits received by a plaintiff from sources other than the defendant do not diminish the defendant's liability. That is definitely true of first party insurance benefits. If they reduce the damages, the defendant would be the ultimate beneficiary, not the plaintiff. Collateral benefits certainly do not increase the defendant's liability, as answer (c) suggests.

Waters v Blackshear

battery requires actor intended to cause harmful/offensive contact; actor need not intend harm suffered if actor intended the contact.

Battery: difference of blowing smoke in someones face vs. showing them porn?

blowing smoke = yes, offensive contact that could constitute battery showing them porn = no, because even though it might be "offensive", it is not a physical contact in the first place (in the example where hugging someone and they found it offensive since you were a stranger, it is different because in that case there was contact, even though no harm)

Annabelle is a schoolteacher. She teaches in the sixth grade. Her class is particularly unruly and Annabelle struggles to maintain discipline. One day, one of her students, Brian, deliberately swings a heavy school bag at another student, Christine, injuring her. Which of the following statements is most accurate? (a) Annabelle is vicariously liable to Christine for the harm caused by Brian (b) Annabelle would be liable to Christine because it is her job to teach her students not to attack one another (c) Annabelle would be liable to Christine because of the special relationship between Annabelle and both of the students, Brian and Christine (d) Annabelle would not be liable to Christine because the law imposes no duty on one person to protect another from harm caused by a third

c). Answer (a) is simply wrong: there would be no vicarious liability in this situation. Answer (b) at least looks like direct liability on the part of Christine, but cites the wrong reason for so concluding. Answer (d) is generally correct as a starting point, but it omits to mention that the outcome is different when there is a special relationship between defendant and plaintiff or defendant and assailant.

Brian is knocked from his motorcycle by the negligent driving of Charles, who crashes his car into Brian's motorcycle. Danielle, a pedestrian passing by just feet away from the impact, witnesses the accident. Brian suffers horrific injuries. About ten minutes later, Ann, Brian's wife, returns to her home from a short visit to the local store. She sees Brian lying bleeding in the street. Both Ann and Danielle suffer acute distress and recurring psychological problems. What would the likely outcome be if Ann and Danielle sued Charles in a state that uses the "zone of danger" test to determine liability in cases of negligent infliction of emotional harm? (a) Ann and Danielle would both win (b) Ann would win but Danielle would lose (c) Ann would lose but Danielle would win (d) Ann and Danielle would both lose

c). Danielle was in the "zone of danger" but Ann was not. The odd result is therefore that the stranger who witnesses the accident recovers but the wife who sees the gory aftermath does not.

Question 2 Which of the following situations would be sufficient to support a res ipsa loquitur inference of fault on the part of the defendant? (A) A glass bottle of soda bottled by the defendant explodes in the plaintiff's hand, injuring the plaintiff (B) The plaintiff suffers a broken leg in the defendant's hospital while under general anaesthetic for an operation on her arm (C) The spare tire falls from beneath defendant's truck and crashes through the windshield of anestheticplaintiff's car (D) A bottle of wine lands on plaintiff's head as she walks past defendant's hotel, which has many balconies

c). In (a), the fact of the explosion is not sufficient in itself to raise an inference that it was caused by the defendant bottler, rather than by someone else such as the bottle manufacturer, distributor, retailer, etc. Similarly, in (b), the fact of the injury would not be enough to indicate who was responsible for plaintiff's injury. Unless the defendant would be vicariously liable for every single person who might have come into contact with plaintiff while she was unconscious, a res ipsa loquitur inference would be unsound. (But see Ybarra v. Spangard, 154 P.2d 687 (Cal. 1944).) Example (d) is like the classic case of Byrne v. Boadle, but the circumstances make it much more likely that the fault was that of one of the guests (for which defendant would not be responsible) than defendant itself. The accident in (c) would be unlikely to occur in the ordinary course of things without fault on the part of the truck owner.

Anita is injured in an accident caused by the negligence of Barbara, Christine and Diana. She suffers $100,000 worth of harm. She sues Barbara, Christine and Diana. Anita settles with Barbara for $10,000, but proceeds to trial against Christine and Diana. The court determines that Barbara was 40% responsible, Christine 40% and Diana 20%. (In other words, Barbara settled for less than the court ultimately decided was her fair share.) What should Christine's liability be in a state that has a system of several (not joint and several) liability? (a) $90,000 (b) $60,000 (c) $40,000 (d) $0

c). Under a system of several liability, a defendant's proportionate share of liability can never exceed his or her share of responsibility. Answer (a) would be the position under joint and several liability, where Christine and Diana would be jointly and severally liable for the amount not already paid by Barbara. Answer (b) makes Christine and Diana jointly and severally liable for the proportion of Anita's harm for which Barbara was not responsible, which is a mix of several and joint and several liability. There is no basis for answer (d), complete immunity.

cause in fact = _____ = ______

cause in fact = legal cause = but for test

indivisible harms (concerted action)

concurrent tortfeasors, each of whom contributed inseparably to the harm

what are the two defenses of consent?

consent + person or property

Brower v. Ackerley

contact must be imminent for it to be assault

In cases about the liability of municipal and state governments, a distinction is often made between "discretionary" and "ministerial"governmental acts. Which of the following most accurately defines a "ministerial" governmental act? (a) An act performed by a government minister (b) An act subject to review by a government minister (c) Conduct involving the exercise of reasoned judgment (d) Conduct requiring adherence to a governing rule, with a compulsory result

d). "Ministerial" acts are not so called because they have anything to do with government ministers, so answers (a) and (b) are wrong. Answer (c) defines a "discretionary" governmental act.

In most cases, a threat of violence made over the telephone cannot constitute an assault. Which of the following most accurately explains why not? (A) Because a reasonable person would not feel afraid as a result of hearing such a threat (B) Because the defendant must be in the same place as the plaintiff for there to be an assault (C) Because the plaintiff would have no reason to suppose that the defendant would follow through with the threat (D) Because the violent contact threatened by the defendant would not be imminent

d). Although the threat in these circumstances might seem very real, it is at most a threat of harm at some unspecified future time. Unless there is something to indicate that the threat could immediately be put into effect by battery of the plaintiff by the defendant, then there is no assault.

Andrew is walking along the beach when he sees Stevie in the water, her arm in the air. He looks more closely and sees that she is drowning, not waving. He smiles enigmatically and waves back before walking off up the beach. Which of the following statements is correct? (a) Andrew owed a duty to try to save Stevie because it was foreseeable that she would drown if he made no effort to do so (b) Andrew would only owe a duty to try to save Stevie if he had some special ability, such as being a strong swimmer or having lifeguard training (c) In no circumstances would Andrew owe Stevie a duty because the law does not require one person to take positive steps for the safety of another (d) Andrew would owe Stevie no duty unless there was some special relationship between them.

d). Answer (a) is the opposite of the true position, answer (c) goes too far in saying that there could never be a duty. Answer (b) raises plausible-sounding but irrelevant matters.

In cases involving birth defects caused by diethylstilbestrol (DES), several states adopted a "market share" theory, by which manufacturers of DES pay compensation in proportion to their share of the market for DES. Which of the following most accurately describes the reason for adopting a "market share" approach? (a) The plaintiff cannot discharge the burden of proving which manufacturer's DES his/her mother took while pregnant (b) It would be unfair to the plaintiff to require proof of causation in the ordinary way (c) The defendant manufacturer cannot prove that the plaintiff's mother did not take the DES that it manufactured (d) Neither plaintiff nor defendant can discharge the usual standard of proof but it is certain that the defendant manufacturer's DES caused some harm to some child, if not the plaintiff

d). Answers (a), (b) and (c) are correct but only partial answers.

Alexander suffers a single injury as a result of separate acts of negligence by Byron and Charles in a state that has not modified the rules of joint and several liability. Damages are assessed at $100,000 . Alexander sues Byron and wins a judgment for $100,000 but Byron is bankrupt and cannot pay Alexander anything. Alexander later sues Charles. Which of the following outcomes will occur? (A) If Alexander establishes that Charles is at fault, judgment will be for nominal damages only (i.e., effectively zero) because Alexander has already won judgment against Byron for the whole of his loss (B) If Alexander establishes that Charles is at fault, judgment will be for nominal damages only (i.e., effectively zero) because Alexander has already won judgment against Byron for the whole of his loss (C) If Alexander establishes that Charles is at fault, he will win a judgment for whatever sum he is unable to recover from Byron (D) If Alexander establishes that Charles is at fault, he will win a judgment for $100,000

d). At one time, the correct answer would have been (b) because Charles's liability would have been discharged by the judgment against Byron, even if Alexander was unable to collect on that judgment. The rule is now that judgment against one tortfeasor does not bar suit against another unless the first judgment is satisfied. Because none of the first judgment against Byron has been satisfied, Alexander can recover in full from Charles. Answers (a) and (c) sound plausible but are inconsistent with the concept of joint and several liability.

Brian is knocked from his motorcycle by the negligent driving of Charles, who crashes his car into Brian's motorcycle. Danielle, a pedestrian passing by just feet away from the impact, witnesses the accident. Brian suffers horrific injuries. About ten minutes later, Ann, Brian's wife, returns to her home from a short visit to the local store. She sees Brian lying bleeding in the street. Both Ann and Danielle suffer acute distress and recurring psychological problems. What would the likely outcome be if Ann and Danielle sued Charles in a state that uses the principles established in Dillon v. Legg, 441 P.2d 912 (Cal. 1968) to determine liability in cases of negligent infliction of emotional harm? (a) Ann and Danielle would both win (b) Ann would win but Danielle would lose (c) Ann would lose but Danielle would win (d) Ann and Danielle would both lose

d). Dillon v. Legg allows recovery to a plaintiff who: (1) is near the scene of the accident; (2) suffers shock from a direct emotional impact as a result of contemporaneous sensory observance of the accident, and (3) is closely related to the victim who suffers physical harm. Neither Ann nor Danielle satisfies all three requirements. Danielle satisfies (1) and (2) but not (3). Ann satisfies (3) but not (1) and (2). Thus, although Dillon v. Legg is generally regarded as being more generous a rule than the "zone of danger" rule, the comparison between this answer and the answer to Q 76 shows that it can operate more restrictively.

Arkady's ship crashes into the Riverwalk Shopping Center as a result of negligence on Arkady's part. None of the shops is damaged, but the elevator shafts and common spaces at one end of the shopping center are severely damaged. Because of the extensiveness of the damage, the entire shopping center is closed down until repairs are completed. All of the shops in the shopping center lose business as a result of the closure. The shop-owners sue Arkady. Which of the following most accurately states the likely outcome of the shop-owners' lawsuits against Arkady in a jurisdiction that applies the "bright line" rule from Robins Dry Dock & Repair Co. v. Flint, 275 U.S. 303 (1927)? (In fact, the shop-owners' claims would be governed by general maritime law because they would be within the court's admiralty jurisdiction. Robins Dry Dock established the "bright line" rule as part of general maritime law.) (a) All of the shop-owners would recover their losses, which were directly caused by Arkady's negligence (b) Only those owners of those shops that were within the "zone of danger" posed by Arkady's ship could recover damages for their losses (c) All of the shop-owners could recover their losses because they were a finite, determinable class of plaintiffs who would likely suffer economic loss as a result of Arkady's negligence (d) None of the shop-owners would recover their losses because none suffered physical damage to their property

d). The "bright line" rule denies recovery to anyone who suffers a purely economic loss without physical injury or property damage. Some of the concepts used in the other three answers are used in economic loss claims in some jurisdictions, some (such as the "zone of danger") are not, but none of them applies under the "bright line" rule.

Anita is injured in an accident caused by the negligence of Barbara, Christine and Diana in a state that has not modified the rules of joint and several liability. She suffers $100,000 worth of harm. She sues Barbara, Christine and Diana. Anita settles with Barbara for $20,000, but proceeds to trial against Christine and Diana. The court determines that Barbara was 40% responsible, Christine 40% and Diana 20%. (In other words, Barbara settled for less than the court ultimately decided was her fair share.) What impact should the settlement of $20,000 already made by Barbara have on the liability of Christine and Diana under the "comparative share" rule? (The "comparative share" rule is preferred by the Restatement (Third) of Torts, Section: 16). (a) Anita should be able to recover $100,000, the full amount of the harm she suffered (b) Anita should be able to recover $80,000, the amount of the $100,000 loss that has not already been satisfied by Barbara (c) Anita should be able to recover $66,667, because she is recovering from two of three jointly and severally liable defendants (d) Anita should be able to recover $60,000, the proportion of Anita's loss for which Christine and Diana were liable

d). The "comparative share" rule produces the result that the non-settling defendants do not end up paying more than their comparative share (60% between them) just because Anita and Barbara settled. In this case, the result would be that Anita would end up with only $80,000, but that would have been caused by her decision to settle for only $20,000 with Barbara, who was 40% responsible.

difference of defense and retaliation

defense = acceptable, its to prevent another act from happening retaliation = not acceptable, after the fact you use force against the person

two types of transfer of intent

designed to facilitate plaintiff's recovery in assault and battery cases Between Torts/Between Parties

Hall v McBryde - main takeaway

doctrine of transferred intent allows defendant's intent to cause harmful/offensive contact to one be transferred to another

what happens if you are unaware of a statute and then violate it?

doesn't matter - we all have a duty to know they exist

Prima Facie elements of Duty of Reasonable Care

duty of care, breach of DOC, causation (in fact & proximate), damages.

when is it always okay for someone to trespass?

exception is when someone is trying to avoid danger; when it is necessary to avoid serious harm (however the trespasser can still be liable for damages to the property while they are running through - ex: if they trample flowers while running through the property)

express v implied consent

express consent is when you state you consent to something; implied consent is when your actions show you were complying with something

What are some examples where courts usually reject Attractive Nuisance doctrine?

for swimming pools, ponds, and other bodies of water

What is pure emotional harm

harm that was NOT preceded by a physical injury to the person suffering the harm (NIED)

creasy v rusk rationales?

i. 5 Public Policy rationales: 1. Allocates losses between two innocent parties to the one who caused or occasioned the loss. 2. Provides incentive to those responsible for people with disabilities and interested in their estates to prevent harm and "restrain" those who are potentially dangerous. 3. Removes inducements for alleged tort-feasors to fake mental disability in order to escape liability. 4. Avoids administrative problems involved in courts and juries attempting to identify and assess the significance of an actor's disability. 5. Forces persons with disabilities to pay for the damage they do if they "are to live in this world"

Walmart Store v Wright - from Ellies notes

i. A company's own rules or policies show the standard of care the actor has chosen to impose on itself ii. Company rules or policies may represent care > a reasonable care standard would require iii. Violation of one's own safety standards is not necessarily equivalent to violation of the reasonable person standard

Peterson v Taylor --- use of an expert witness?

i. A jury uses its own knowledge of children when it applies the child's standard of care ii. But must base its fact-finding on evidence that informs it of the attributes of the particular child whose conduct it must evaluate

do you have to be an expert for custom testifying?

i. A witness can testify about custom if qualified by knowledge and experience, without being an expert - just have to be able to determine if it is a matter of fact not opinion

Transfer of Intent - Between Torts

i. Allows plaintiff who suffers harmful/offensive contact to recover for battery even if defendant only intended an assault; allows plaintiff who suffers apprehension of imminent harmful/offensive contact to recover for assault even if defendant only intended battery.

Woodard v. Turnipseed

i. An actor is privileged to use reasonable force , not intended or likely to cause death or serious bodily harm, to prevent or terminate another's intrusion upon the actor's land or chattels, if: 1. The intrusion is not privileged or the other intentionally or negligently causes the actor to believe that it is not privileged 2. The actor reasonably believes that the intrusion can be prevented or terminated only by force used 3. The actor has requested the other desist and he did not listen, reasonably believes the request will be useless, or substantial harm will be done before it can be made

3 things from Ellie's outline about Young v Warren:

i. An assault on a third party in defense of a family member is privileged only if the defendant had a well-grounded belief that an assault was about to be committed by another on that family member ii. The privilege protects the defendant from liability only to the extent that the defendant did not use more force than was necessary or reasonable iii. The necessity for defense must be immediate and attacks made in the past and threats for the future will not justify the privilege

why didn't the nurse win in Creasy v. Rusk if we generally say mentally disabled people are to be held to the same SOC as regular people?

i. Courts may moderate this rule in particular circumstances, such as a setting in which the mentally disabled person's opponent ought not to be owed a duty by the mentally disabled person

Lyons v. Midnight Sun Transportation Services

i. Even if D was negligent, D's act must separately be proved to be a cause-in-fact ii. D's negligence may not have been a necessary event in producing harm iii. A non-negligent act may be a cause-in-fact iv. Accidents that are not result of fault are sometimes called "mere" accidents

Shull v. B.F. Goodrich Co.

i. Exclusive control element of the res ipsa loquitur doctrine can be satisfied even if the defendant is not in control of the injury-causing instrumentality at the time of injury, as long as defendant was in control of it at a time when negligence related to injury was likely to have taken place ii. Some states require the plaintiff to show that the defendant has more access to evidence of the explanation for the malfunction than the plaintiff

Cay v State of Louisiana from Ellies slides

i. Must reach conclusions about 2 topics 1. What happened to the decedent 2. Whether the alleged negligent conduct by the defendant was a necessary element for what happened to the plaintiff ii. There are infinite # of causes-in-fact iii. Cause-in-fact means that harm would not have occurred but for act in question iv. Cause-in-fact may be inferred by considering likely ways harm could have come about, likelihood of each course of events, and role act in question played in each

Byrne v Boadle

i. Proof that an injury occurred in circumstances that make it very likely that it was caused by a defendant's negligence can be treated as equivalent to proof that a defendant was negligent

Parrot v. Wells Fargo

i. Reasonable Prudence depends greatly on Knowledge an actor Possesses or Should possess ii. Negligence = "the omission to do something which a reasonable man, guided by those considerations which ordinarily regulate the conduct of human affairs, would do, or doing something which a prudent and reasonable man would not do"

Modern Approaches to duty to people on land

i. Rejection of the 3-category system in favor of general duty of care to all entrants 1. Other states combined licensee & invitee into a 2-category system (lawful & unlawful) i. Open and obvious doctrine can be a shield to landowner liability but not a complete shield (distraction exception imposes liability when landowner failed to remedy harm & business invitee distracted) 1. Even if danger was open and obvious, jury must also find out whether the landowner breached a duty by allowing the danger to exist. ii. Landowners held to reasonable care standard 1. Continuing duty to make repairs and make premises habitable and shit.

Dover Elevator Co v. Swann

i. Some courts refuse to allow a res ipsa loquitur instruction where a plaintiff 1. Has a factual theory of what the defendant did that was negligent and 2. Offers evidence to prove that the defendant was negligent in some particular way a. E.g. expert testimony

Dover Elevator Co. v. Swann

i. Some courts refuse to allow a res ipsa loquitur instruction where a plaintiff 1. Has a factual theory of what the defendant did that was negligent and 2. Offers evidence to prove that the defendant was negligent in some particular way a. E.g. expert testimony

5 elements of statutory interpretation in Negligence Per Se

i. Statutory Interpretation 1. Variations in the treatment of statutes 2. Every word in a statute must be given effect and meaning 3. Examine statute as whole 4. Avoid strict, literal, or selective reading 5. Words have plain and ordinary meaning

Learned Hand Formula

i. The "Hand Formula" 1. B < PL a. B = burden of prevention or avoidance b. P = probability of loss c. L = magnitude of loss that would be avoided with the possible prevention or avoidance d. PL = benefit of taking precaution 2. An actor is negligent if B is less than P

4 things you need for a plaintiff to recover damages to show a tort has been committed: (prima facie case)

i. The defendant owed the plaintiff a duty to act in a certain way (duty) ii. The defendant breached the duty by failing to act as well as the duty required (breach) iii. The defendant's conduct caused some (causation) iv. Harm to the plaintiff (damages)

2 modern Res Ispa Loquitor Requirements

i. The requirement that accidents like the one that occurred are usually the result of someone's negligence ii. The instrumentality that caused the harm was in the actor's control at the time of the negligent act

two main elements of ipsa loquitur

i. The requirement that accidents like the one that occurred are usually the result of someone's negligence ii. The instrumentality that caused the harm was in the actor's control at the time of the negligent act

what does Martin v Herzog tell us about negligence per se?

i. This means there is no need for evidence related to costs and benefits of taking precaution or reasonableness of behavior ii. Proof of causation may be a separate issue

two ways one can be reckless?

i. To be reckless, conduct must involve an unreasonable or intentional disregard of a risk that presents a high degree of probability that substantial harm will result ii. Recklessness can also include inaction where there is a duty to act

5 elements of negligence per se

i. Type of plaintiff ii. Type of harm iii. Type of defendant iv. Specific v. general conduct v. DO NOT SAY it's unreasonable to violate statutes

Wawanesa v Matlock - takeaways

i. Unless legislation was intended to prevent the type of harm that occurred, proof of statutory violation has no relevance in a tort case ii. Statutory Interpretation 1. Variations in the treatment of statutes 2. Every word in a statute must be given effect and meaning 3. Examine statute as whole 4. Avoid strict, literal, or selective reading 5. Words have plain and ordinary meaning

explain the difference of joint and several and SEVERAL

if Ann is struck by a car driven by Bob who was served alcohol in Charlotte's bar, then both Bob and Charlotte may be held jointly liable for Ann's injuries. Bob was 90% at fault and Charlotte 10%. ($10 million total). Under several liability: Bob has to pay 9 million. If Bob has no money, then Ann only gets the $1 million from Charlotte. Under Joint and Several: Ann could recover from either defendant the full amount

what does a not greater than system mean?

if you were responsible for a larger proportion than either of the defendants that harmed you, you cant get ANYTHING

people on your land from most to least rights

invitee > licensee > trespasser

for which type of guest does a landowner owe a duty to repair known defects on the land?

invitees only

when are you subject to liability for trespass

irrespective of whether they thereby cause harm to the person/land , if you satisfy one of the three elements of trespass you're still held liable

under comparative share, what happens if one defendant settles for MORE than they were supposed to based on their share?

it doesnt matter; the liability of the non-settling defendants is not affected by the first settlement; they're still liable for the portion they would have been liable for if the first person never settled

can you prove a defense for NPS by saying you acted reasonably?

maybe; the jury would have to decide that they think you really did act reasonably

battery - what two things must you do

must not only intend to ACT but to act FOR the purpose of inflicting a harmful or offensive contact on the plaintiff

Alternatives to "But-For" test

necessary v sufficient multiple sufficient causes concert of action multiple defendants and liability alternative liability market share liability

NIED

negligent infliction of emotional distress;

can you use self defense on someone if you have a chance to retreat?

no (unless the state doesnt require retreat before force)

when is contact intentional

no contact is intentional if its not the result of a voluntary act (ex: if you faint on someone, or if someone pushes you into someone else)

conditional threats (in assault)

not enough to satisfy the mens rea of assault because they're not immediate.

evidentiary approach of the impact of future earnings - inflation and productivity permits consideration of each issue 3 methods:

offset present value method- subtracted the estimated inflation rate from the discount rate to reach the "real" rate of interest. second variant was developed by the Alaska Supreme Court in which lost future earning capacity of the victim is calculated over his work-life expectancy. "total offset method" which, rather than discounting the award to present value, assumes that the effect of the future inflation rate will completely offset the interest rate, thereby eliminating any need to discount the award to its present value.

what does nuisance require?

ongoing interference - one single event cannot create a nuisance (must be ongoing and substantial)

what degree of force can you use for self defense

only force that is equal to or lesser than the threat that is coming at you

what is the distraction exception an exception to

open and obvious rule

several liability

parties liable for only their portion of the harm they caused (proportionate liability) ex: loan agreements with multiple banks, each bank only liable for THEIR part of loan

Recklessness tort

person acted without regard for other person

under traditional But For test who has burden of proof?

plaintiff

joint and several (46 out of 50 states)

plaintiff can recover entire amount of judgment from either party no matter what % liable they are -if person who got harmed only goes after one, its up to the defendants to sort out what proportions they owe; and if one party refuses to pay then the other party can completely pay it off

McCarty v. Pheasant Run

rape case plaintiff = mccarty, defendant = pheasant run 1981 mccarty checks into pheasant run hotel for sears business meeting. goes for dinner, comes back and is attacked, fends off attacker. she had left sliding glass door unlocked and only put the safety chain on. 1. duty - keep guests safe 2. breach - no breach because of reasonable man and hand formula 3. causation - yes, yes (yes foreseeable if someone were to enter room they might harm guest) 4.damages - yes hand formula as it applies: P*L > B = breach 1% * 1,000,000 = 10,000...but here B is greater than 10,000 (guards salary + locks...) so no breach It is not efficient to prevent all accidents.

how might one take the holding as Summers v Time as too broad?

some think you apply it to shift burden to defendants ANY time there are multiple defendants; BUT all its saying is if the bit for test is NOT satisfied, THEN we shift it to the defendants

TJ Hooper

sometimes following a custom of an industry is not good enough; one must use reasonable care i. Proof of compliance with industry custom is relevant to the question of reasonableness of conduct, but is not controlling on that issue ii. Rule: Compliance with custom is proofof reasonable care, but not the deciding factor

what happens if you try to help someone even though they protest and then there is some harm resulting from that help?

still a battery - shouldn't have done it if she was saying no because clearly she finds the contact to be offensive

compare Contributory negligence, Comparative negligence, and strict liability

strict liability: even if the plaintiff was a part of the "but for" cause, the defendant is still fully liable comparative negligence: if the plaintiff was contributory as a "but for" negligence cause, their damage awards will be reduced in relation to how negligent they were in the accident contributory negligence: if the plaintiff was contributory in ANY WAY, they don't get anything (easiest on defendant)

what does someone NOT need to show when making a claim for the attractive nuisance doctrine?

that the child was lured onto the property BECAUSE of the attractive nuisance

Res Ispa Loquitur means?

the thing speaks for itself - circumstantial evidence sometimes facts can be inferred from other facts that prove negligence

joint liability

there is more than one party responsible ---- each liable up to full amount of relevant obligation ex: if one married party disappears the other party is liable for full amount of debt so if there are three people liable for something and ONE pays full amount then bank cant sue the other two people for full amount, the other two people are off the hook

summers v Tice

told us that if we can't satisfy the but for causation test, we shift the burden of proof to the two defendants in proving it wasn't them; if neither can prove, we hold them both liable

T/F? when doing a negligence per se question - make sure the thing the statute is meant to protect is the thing that actually happens as a direct cause of action

true

Pro Tanto v. Comparative Share

under comparative share, if one defendant has already settled for less than their portion, the other two defendants aren't going to have to pay more between the two of them than their own portion of the damages under pro tanto, if one defendant has already settled for less than their fair share, the other two defendants are going to have to split everything thats left between them

presumption states - NPS

violation of a statute that establishes a standard of care in itself establishes negligence unless the violator provides some form of a proof of excuse

requirements of negligence per se

violation of statute harm of the type that the statute intended to prevent harmed person in the class that the statute intended to protect

are tortfeasors liable for later consequences of their torts?

we usually say yes - they are liable for their torts no matter how unexpected - BUT only to an extent...... for example its SO unforeseeable that a freak burn injury would happen later on when a person goes to the hospital d/t something you did

Transferred Intent

when a defendant intends to harm one individual, but unintentionally harms a second person

Divisible Harm

when you can trace a specific harm back to a specific defendant when there are multiple defendants

do you owe duty if there is a special relationship?

yes

What is the objective test to consent?

you consent to a contact, not to the harm itself (consent does not equal consent to ALL harms)


Conjuntos de estudio relacionados

Unit 20. Vocab. A. Choose the correct word.

View Set

Ap Human 3A Quiz (from Barron's 9th Edition)

View Set

ATI Introduction to Health Assessment Test

View Set

Chapter 16 reading quiz questions

View Set

Words of similar meaning - Adjectives

View Set

ALL Chapters 1-5 Strategic Management Exam Practice

View Set

Ch 2 Test Review Criminal Justice Molenar

View Set